kap review 1 Flashcards

1
Q

Admitting the diary when it did not refresh the recollection

A

In this question, the plaintiff in a civil case is attempting to offer information contained within a diary admitted at trial. The plaintiff is attempting to offer the diary because the plaintiff was unable to remember details surrounding air pollution she claims has led to her suit against an industrial facility. The diary is an out-of-court statement, and therefore inadmissible unless a hearsay exception is applicable. Rule 803(5) admits a record that: (1) is on a matter the witness once knew about but now cannot recall well enough to testify fully and accurately; (2) was made or adopted by the witness when the matter was fresh in the witness’s memory; and (3) accurately reflects the witness’s knowledge. Here, the plaintiff was previously aware of these details, but has since forgotten. The plaintiff noted these details in her diary, which she testified to maintaining accurately. The basic elements have been satisfied. Additionally, however, Rule 803(5) lays out what and how the previously known, but currently forgotten, information is introduced. If admitted, the record may be read into evidence, but may be received as an exhibit only if offered by an adverse party. Therefore, the plaintiff should have the diary read into the record, but cannot offer the diary as an exhibit. Answer (D) correctly describes this process.

How well did you know this?
1
Not at all
2
3
4
5
Perfectly
2
Q

Testimony about plaintiff’s reputation in the community as a thief

A

In slander cases, where the defendant makes a statement that the plaintiff has an unsavory character, the plaintiff’s character is considered “in issue” (i.e., an essential element of the claim or defense under the substantive law) in two respects: First, the plaintiff’s actual character will determine whether the defendant was incorrect in his assessment, and thus liable for slander, because truth is a defense. Second, the plaintiff will allege that he has been damaged by the statement, which is another way of saying that his true character has been besmirched; but if the plaintiff actually has a bad reputation anyway, then damages are limited. Thus, in slander cases like the one in this question, character evidence is relevant both to whether the plaintiff has a certain character and to the extent of damages. Under Federal Rule of Evidence 405, when character is “in issue” it can be proved by evidence of reputation, opinion, or specific acts.

How well did you know this?
1
Not at all
2
3
4
5
Perfectly
3
Q

Dying declaration

A

While the declarant does not have to die for a statement to be admissible as a dying declaration under Federal Rule of Evidence 804(b)(2), this statement fails to satisfy that exception for at least two reasons. First, the declarant has to be unavailable, as the dying declaration is one of the “unavailability-dependent” exceptions of Rule 804. Here, the victim testified and so obviously is not unavailable. Second, a dying declaration is admissible only in homicide prosecutions and civil cases. This is a criminal case for aggravated assault.

How well did you know this?
1
Not at all
2
3
4
5
Perfectly
4
Q

The witness has testified that she knows about the defendant’s reputation. The prosecutor has the right to test the basis and adequacy of that knowledge, as well as the nature of the community itself. If the witness answers that she had not heard about the arrest, that admission could indicate that she is not very knowledgeable about the defendant’s reputation in the community, because such an arrest would likely have a negative effect on that reputation. If the witness says that she had heard about the arrest, a negative inference could be raised about the community itself and its view of what it is to be an honest person.

A

The witness has testified that she knows about the defendant’s reputation. The prosecutor has the right to test the basis and adequacy of that knowledge, as well as the nature of the community itself. If the witness answers that she had not heard about the arrest, that admission could indicate that she is not very knowledgeable about the defendant’s reputation in the community, because such an arrest would likely have a negative effect on that reputation. If the witness says that she had heard about the arrest, a negative inference could be raised about the community itself and its view of what it is to be an honest person.

How well did you know this?
1
Not at all
2
3
4
5
Perfectly
5
Q

Daubert test

A

The elements of the Daubert test are: (1) has the theory been tested? (2) general acceptance in the relevant community; (3) peer review regarding the scientific theory; (4) degree or rate of error; and (5) standards. Whether or not there has been a third-party publication of findings is not a factor under the Daubert test.

How well did you know this?
1
Not at all
2
3
4
5
Perfectly
6
Q

Confrontation clause based on Crawford

A

Under the Sixth Amendment’s Confrontation Clause, in a criminal case where the declarant is unavailable, testimonial hearsay statements will be inadmissible unless the defendant is or was given an opportunity to cross-examine the declarant. In determining whether the statement is testimonial in nature, the court will not consider whether the admissibility of the statement offends the interest of justice as a factor.

How well did you know this?
1
Not at all
2
3
4
5
Perfectly
7
Q

hot pursuit search for evidence

A

If police have probable cause, they may search without a warrant when they reasonably believe that waiting to obtain a warrant would result in the imminent flight of the suspect, imminent destruction of evidence, or imminent danger to police or others in the area. When police are in “hot pursuit” of a suspect with probable cause to arrest them, exigent circumstances allows them to enter any home the suspect retreats into without a warrant–even the home of a third party. This fact pattern falls under the “hot pursuit” exigent circumstance exception that would allow a warrantless search.

How well did you know this?
1
Not at all
2
3
4
5
Perfectly
8
Q

The defendant must show that the unreasonable search or seizure violated the defendant’s personal constitutional rights. The defendant may not vicariously assert someone else’s rights.

A

The defendant must show that the unreasonable search or seizure violated the defendant’s personal constitutional rights. The defendant may not vicariously assert someone else’s rights.

How well did you know this?
1
Not at all
2
3
4
5
Perfectly
9
Q

implied easement by necessity

A

The man then conveyed the rear half of the land to the investor, whose land is now landlocked. An easement by necessity is implied for the benefit of the landlocked parcel at the moment of severance of the common ownership. The fact that the neighbor has offered to sell the investor a right of access is irrelevant.

How well did you know this?
1
Not at all
2
3
4
5
Perfectly
10
Q

Restrictive covenant and marketability of title

A

A right held in the land by a third party, such as the right to enforce a restrictive covenant, renders the title unmarketable,

How well did you know this?
1
Not at all
2
3
4
5
Perfectly
11
Q

A deed to a nonexistent grantee

A

A deed to a nonexistent grantee, such as a corporation that has not yet been legally formed, is void. At the time the businesswoman attempted to convey the land to the corporation, the corporation had not yet been legally formed, so the deed was void.

How well did you know this?
1
Not at all
2
3
4
5
Perfectly
12
Q

Deed restraints on transfer

A

Restraints on transfer are void as a matter of law because they are against the public policy of letting landowners sell their property. Consequently, the attempted restraint on transfer was not a title defect that could stop the sale.

How well did you know this?
1
Not at all
2
3
4
5
Perfectly
13
Q

Adverse possession and subsequent transfers

A

The woman acquired title to the farm by adverse possession. The woman’s title was an original title and did not derive from the man’s title. The statute of frauds requires that the conveyance of the farm be in writing. Therefore, the woman’s oral statement was insufficient to release the title to the man, and the woman validly conveyed the farm to her son. Having established title to the farm by adverse possession, there is no requirement that the woman sue to establish title. Therefore, she could convey the farm to her son.

How well did you know this?
1
Not at all
2
3
4
5
Perfectly
14
Q

Voluntariness with police informant

A

Whether a statement is made “voluntarily” depends on the totality of the circumstances which considers the individual characteristics of the defendant (age, sex, race, mental conditions) and the nature of the police conduct. Where a confession is obtained by using a credible threat of force, the courts have held that the circumstances are sufficiently coercive to undermine a claim of voluntariness. Here, the informant was used to convey a credible threat of violence and so the confession was not voluntary.

How well did you know this?
1
Not at all
2
3
4
5
Perfectly
15
Q

May grand jury consider illegally seized evidence

A

The Supreme Court made clear in United States v. Calandra [414 U.S. 338 (1974)] that a grand jury may consider illegally seized evidence in arriving at a finding of probable cause to return an indictment against a defendant.

How well did you know this?
1
Not at all
2
3
4
5
Perfectly
16
Q

Right to a jury trial and to an attorney

A

The right to a jury trial attaches in any criminal proceeding where the defendant faces a potential sentence of longer than six months. Because the maximum imprisonment for the defendant here was seven months, the defendant would be entitled to a jury trial. The right to counsel applies to all defendants charged with an offense for which imprisonment is imposed, whether classified as petty, misdemeanor, or felony. However, if the offense is a misdemeanor, the constitutional right to counsel applies only if imprisonment is actually imposed. An accused charged with a misdemeanor who has not waived the right to counsel and is not represented by an attorney is not subject to imprisonment. Accordingly, the judge’s instruction was incorrect with respect to both the right to a jury trial and the right to counsel, because the defendant was subject to more than six months’ imprisonment and an actual jail term was imposed.

How well did you know this?
1
Not at all
2
3
4
5
Perfectly
17
Q

Breaking

A

To commit a “breaking,” it is not required that a defendant go through a locked door or physically damage the property; any enlargement of an opening will suffice. Thus, the pushing up of the window constitutes a breaking, and the client will not escape liability for burglary on the grounds that he reached in through an unlocked window.

How well did you know this?
1
Not at all
2
3
4
5
Perfectly
18
Q

Conspiracy, withdrawal, and foreseeability

A

The oldest friend’s actions constituted a withdrawal from the robbery, which prevents him from being liable for the subsequent murder. However, his actions would not be a defense to the crime of conspiracy. Conspiracy is an agreement between two or more persons to achieve to achieve a criminal objective. Most states also require an overt act in furtherance of the conspiracy. Accordingly, each conspirator is liable for the crimes of all the other conspirators if the acts were in the furtherance of the conspiracy and foreseeable. Thus, the oldest friend cannot withdraw from the conspiracy because all of the elements have been met. However, his withdrawal would cut off any liability for crimes committed subsequent to that, including the robbery and death of the victim here.

How well did you know this?
1
Not at all
2
3
4
5
Perfectly
19
Q

Right to counsel and identification

A

The right to counsel attaches at the initiation of adversarial proceedings. The Supreme Court has held that the right to counsel attaches when a defendant is arraigned [Moore v. Illinois, 434 U.S. 220 (1978)]. Once the right to counsel attaches, the defendant has a right to counsel at all critical stages of the criminal proceedings, including identifications [Kirby v. Illinois, 406 U.S. 682 (1972); United States v. Wade, 388 U.S. 218 (1967)]. When the suspect was arraigned, his right to counsel attached. However, his counsel was not present at the show-up identification in the husband’s hospital room. As a result, the husband’s identification is inadmissible because the identification procedure violated the suspect’s Sixth Amendment right to counsel.

How well did you know this?
1
Not at all
2
3
4
5
Perfectly
20
Q

Search of bystanders during warrant execution

A

When police execute a search warrant, they are entitled to search the person named in the warrant or the premises identified in the warrant, as well as any other person that is arrested. A person unnamed in the warrant may not be searched merely because of his presence at the search location; there must be an independent justification for the search of a person unnamed in the warrant, such as that person’s arrest or consent. Here, the customer was unnamed in the warrant and there was no independent basis to search him, because he was not arrested along with the barber. For example, the search was not incident to a lawful arrest of the customer. Since the search of the customer was beyond the scope of the warrant being executed at the barbershop, the court will suppress the evidence seized from the customer.

How well did you know this?
1
Not at all
2
3
4
5
Perfectly
21
Q

Failure to deny an allegation in complaint

A

For each allegation, the defendant should specifically admit or deny. If there is a failure to deny, it is deemed admitted.

How well did you know this?
1
Not at all
2
3
4
5
Perfectly
22
Q

Civ Pro Numbers Game 1
6
10
14
21

A

6 - need at least 6 jurors to start and finish a civil trial
10 - depositions per party as a matter of right
14 - days from the last responsibe
14 - days default duration of a TRO
21 - days to amend a complaint as a matter of right
21 - days to respond to a complaint
21 - days to fix a Rule 11 problem before sanctions

How well did you know this?
1
Not at all
2
3
4
5
Perfectly
23
Q

Civ Pro Numbers Game 2
25
28
30
60
90

A

25 - interrogatories as a matter of right for each party
28 - days after losing a lawsuit to move for a new trial
28 - days for renewed motion for judgment as a matter of law (Rule 50)
30 - days to appeal after losing at trial
30 - days to remove after removal becomes possible
60 - days to answer a complaint when defendant waives service of process
90 - days to answer a complaint when defendant waives service of process and is foreign
90 - days to serve the complaint after filing the lawsuit
90 - days to disclose experts who will be used at trial

How well did you know this?
1
Not at all
2
3
4
5
Perfectly
24
Q

Objections to the jury charge

A

:Federal Rule of Civil Procedure 51(c)(2) governs the timeliness of objections to the jury charge. Objections have to be made both: (1) before closing arguments; and (2) before the instructions are read to the jury. When, as here, the court informed the parties of the proposed instructions before the jury was charged and final jury arguments were given, the party had to object at that time. The plaintiff in this case did not object until after the charge had been given and thus the objection came too late.

How well did you know this?
1
Not at all
2
3
4
5
Perfectly
25
Q

Dismissal for failure to prosecute

A

If a plaintiff fails to prosecute his or her action or to comply with the Federal Rules or a court order, a defendant may move to dismiss [Fed. R. Civ. P. 41(b)]. A dismissal for failure to prosecute is a dismissal with prejudice, unless the dismissal order states otherwise. Congress has given to the federal courts subject-matter jurisdiction over appeals from all final decisions of the federal district trial courts [28 U.S.C. Sec. 1291]. A final judgment is generally defined as one that disposes of all issues as to all of the parties. Here, the dismissal with prejudice would act as a final order, giving the court of appeals jurisdiction over the case.

How well did you know this?
1
Not at all
2
3
4
5
Perfectly
26
Q

Juror’s implied bias

A

A lawyer may ask the court to excuse any juror for cause. There are three categories for such challenges: (1) general disqualification (such as a felony conviction or some other inherent disqualification); (2) implied bias (the attorney believes the juror may be biased); and (3) actual bias (the juror indicates during voir dire that he or she would decide the case based on predetermined beliefs or values). The first three jurors have been excused based on actual bias. The fourth juror has exhibited an implied bias. As such, the attorney may request that this juror be excused as well.

How well did you know this?
1
Not at all
2
3
4
5
Perfectly
27
Q

Motion for JMOL

A

Within 28 days after the entry of judgment, a party who has timely moved for judgment as a matter of law (JMOL) may serve a motion to set aside the verdict and any judgment entered on the verdict [Fed. R. Civ. P. 50(b)]. However, the court may not entertain a renewed motion for JMOL unless a motion for JMOL was made during trial.

How well did you know this?
1
Not at all
2
3
4
5
Perfectly
28
Q

Search of the entire house incident to lawful arrest

A

A search incident to a lawful arrest may include a cursory scan or “protective sweep” of adjoining rooms, but in order to search the entire domicile (as here), the officers must have a reasonable suspicion that an armed accomplice is on the premises [Maryland v. Buie, 494 U.S. 325 (1990)]. Here, there is nothing in the facts to suggest that the police could have had a reasonable suspicion that an armed accomplice might, in fact, have been present in the house. Therefore, the constitutional search was limited to a cursory scan of adjoining rooms. The search of the “entire” house violated the doctor’s Fourth Amendment rights. (Had the search been lawful, the evidence would have clearly qualified for the “plain-view” exception to the warrant requirement.)

How well did you know this?
1
Not at all
2
3
4
5
Perfectly
29
Q

Plain view doctrine

A

Two requirements must be met in order for the plain-view doctrine to justify a warrantless seizure of property, as follows: (1) the police must be lawfully positioned; and (2) it must be immediately apparent that the evidence is incriminating [Coolidge v. New Hampshire, 403 U.S. 443 (1971)]. Here, the spot of blood satisfies the elements, but the discovery of the knife does not.

How well did you know this?
1
Not at all
2
3
4
5
Perfectly
30
Q

Accomplice and arrest warrant

A

When police execute an arrest warrant at a suspect’s home, the arrest warrant provides implicit authority to enter the home and search for the suspect to effectuate the arrest if police reasonably believe the suspect is in the home and refuses to answer the door. If the suspect is the only person police believes is in the home the scope of such a search is limited to looking for the suspect, and it terminates when the suspect is located and arrested. However, when police have reasonable suspicion that others in the home may pose a threat to them, they are authorized to conduct a cursory “protective sweep” of the home to rule out this risk. This allows them to look in locations where such a person may be, and any contraband that comes into plain view while acting within that limited scope may be seized. In this question, the officers entered the home lawfully to execute the arrest warrant. However, the reference to an “accomplice” to a series of armed robberies indicates the officers reasonably believed that: (1) the accomplice would be in the home with the suspect; and (2) the accomplice could be armed and threaten the officers in an effort to prevent their efforts to arrest the suspect. Accordingly, they were authorized to conduct a cursory protective sweep. Because the officer who found the contraband was looking in a place where the accomplice could be hiding, the contraband was discovered within the scope of the protective sweep. Because what the officer saw created immediate probable cause it was contraband, the seizure of the cocaine was lawful pursuant to the plain view doctrine.

How well did you know this?
1
Not at all
2
3
4
5
Perfectly
31
Q

Robbery and the use of force

A

Robbery at common law was the commission of a larceny by the use of force or threat of force beyond the amount of force needed to merely take the property. The common elements of the offense are: (1) the taking of the property of another (2) from his or her person or in their presence (3) by violence, intimidation or threat (4) with the intent to deprive them of it permanently. A perpetrator is not required to use significant force, or extreme threats, in order to commit a robbery. All that is required is the amount of violence or fear necessary to cause the victim to give up his or her possessions. But it is also required that the force or threat be part of the theft transaction, which means if it is necessary to use force or threat to effectuate the unlawful taking and carrying away the force element of robbery is satisfied. Here the facts indicate defendant pushed the victim in order to effectuate the unlawful taking, and thus while he was doing so to “get away” he was also doing so to commit the larceny. Therefore he is guilty of robbery.

How well did you know this?
1
Not at all
2
3
4
5
Perfectly
32
Q

Enhanced sentence burden of proof

A

The Supreme Court established that due process requires the prosecution (state) to prove any fact triggering an enhanced sentence beyond a reasonable doubt, and if the defendant is entitled to a jury trial (and does not waive that right) a jury must make the finding that this enhancement was proven beyond a reasonable doubt. All other answers are inconsistent with this due process standard.

How well did you know this?
1
Not at all
2
3
4
5
Perfectly
33
Q

Merger of solicitation into conspiracy

A

Conspiracy is an agreement, express or implied, between two or more people to commit a crime or a lawful act by unlawful means (like fraudulently obtaining a mortgage). Conspiracy normally requires proof of an overt act that establishes planning to commit the crime had actually begun. Although the defendant did solicit his co-conspirator to commit the murder, that solicitation merges into the conspiracy and is not a viable offense once that merger occurs. All the elements of conspiracy are therefore established by these facts. Unlike solicitation and attempt, conspiracy does not merge with the completed crime, even if that crime is the direct object of the conspiracy. However, the solicitation of the murder is evidence that makes the defendant an accomplice to the murder: he intended that crime occur and his solicitation was the act of facilitation. This means the defendant is guilty of the murder as if he committed it. the defendant is also guilty of the murder through conspiracy liability: co-conspirators are guilty of all crimes that are foreseeable outgrowths of the agreement in furtherance of the agreement. Because the murder was both foreseeable and in furtherance the defendant is guilty of that crime as if he committed it.

How well did you know this?
1
Not at all
2
3
4
5
Perfectly
34
Q

Deposing a non-party witness

A

Under Fed. R. Civ. P. 30(b), a party who wants to depose a person by oral questions must give reasonable written notice to every other party. Each party is entitled to attend any deposition and ask questions on cross-examination.

How well did you know this?
1
Not at all
2
3
4
5
Perfectly
35
Q

Product liability defendants

A

In a products liability case, commercial suppliers at all levels of the distribution chain and those in the market of selling the product–including the manufacturer, wholesaler, and retailer–are considered proper defendants. However, occasional sellers and those supplying services are not.

How well did you know this?
1
Not at all
2
3
4
5
Perfectly
36
Q

The concerned citizen was acting under the direction of a government agent

A

The concerned citizen was acting under the direction of a government agent, and therefore, her search of her friend’s computer was governed by the Fourth Amendment. The concerned citizen obtained permission to use the computer to check her email. When the concerned citizen’s friend agreed, it was for that limited use. Consent as a defense to a charge of illegal search has three elements: (1) the consent must be unforced and the result of an informed decision; (2) the search must not exceed the scope of the consent; and (3) the consenting party must have authority to consent. In this case, the friend had no reason to know that a search was going to be conducted, so her consent was by trick. Furthermore, the consent to use the computer was for the concerned citizen to check email. The citizen’s search for secret files exceeded the scope of her permission to use the computer. As such, the search was illegal, and the evidence will be suppressed.

How well did you know this?
1
Not at all
2
3
4
5
Perfectly
37
Q

Unilateral theory of conspiracy

A

The Model Penal Code applies the unilateral theory of conspiracy, which holds that a defendant can be found guilty of conspiracy regardless of whether other persons were true to their words of intent. As such, this answer choice is correct.

How well did you know this?
1
Not at all
2
3
4
5
Perfectly
38
Q

Duty to charge reasonable fees

A

The Georgia Rules of Professional Conduct mandate that a lawyer’s fee be reasonable, and several factors are enumerated that help determine reasonableness. One factor is the customary fee in the local area. Under that factor, the lawyer’s fee here seems unreasonable because it is double what lawyers customarily charge for representation of creditors. However, another factor is the likelihood that, if the lawyer accepts this employment, she will be precluded from accepting other legal employment. These facts clearly show such a situation. In this case, the lawyer would have preferred to work for the debtor and, if she worked for the creditors, she would lose her ability to do so. Note that another factor that may affect the reasonableness of a fee is the lawyer’s skill. Here, the lawyer has a reputation for being “the best,” and that also helps to justify this higher-than-normal fee. Under the totality of the circumstances, the lawyer’s fee is reasonable, and she will be able to recover the entire fee.

How well did you know this?
1
Not at all
2
3
4
5
Perfectly
39
Q

Lawyer referral fees to other lawyers

A

The Georgia Rules of Professional Responsibility distinguish between a division of fees between two lawyers not in the same firm who work on a case together and referral fees (payments not based on the referring lawyer’s work on the matter). Referral fees are not permitted in Georgia unless each lawyer assumes joint responsibility for the representation and the client gives written consent. As such, the two lawyers’ conduct violates the Georgia Rules of Professional Conduct.

How well did you know this?
1
Not at all
2
3
4
5
Perfectly
40
Q

Disclosure to prevent harm

A

The Georgia Rules of Professional Conduct are somewhat different and, in this situation, more permissive than the Model Rules. They allow a lawyer to disclose information he reasonably believes necessary to prevent a substantial financial loss to a third party, even if the client has already taken the harmful action. The key is that the disclosure is permissible if the loss has not yet occurred. Here, although the client’s death makes it too late for the lawyer to persuade the client to right his own wrongs, the lawyer’s disclosure can prevent the losses by informing the victims what is about to happen to them before the estate’s funds are no longer available. Therefore, this is the correct answer under the Georgia Rules.

How well did you know this?
1
Not at all
2
3
4
5
Perfectly
41
Q

Conducting unrelated business with a client

A

Under the Georgia Rules of Professional Conduct, a lawyer may conduct business with a client only if the terms of the arrangement are fully disclosed to the client in writing, the client is given a reasonable amount of time to consult an outside lawyer, and the client gives his informed written consent. In this case, although the lawyer explained the terms to the real estate developer, the explanation was oral, not written. Therefore, even though there does not appear to be any abuse and all parties are satisfied with the transaction, the lawyer still violated the Georgia Rules of Professional Conduct.

How well did you know this?
1
Not at all
2
3
4
5
Perfectly
42
Q

Content neutrality when renting facilities

A

The city is infringing upon freedom of speech because it will only rent the facilities to groups who engage in non-religious speech and meet for non-religious purposes. The Supreme Court has struck down such a practice in Widmar v. Vincent [454 U.S. 263 (1981)]. In the Widmar case, the Court held that a state university ban on the use of its public meeting facilities for prayer and religious discussion by student groups violated the “fundamental principle that a state regulation of speech should be content-neutral.” Government regulations that discriminate against the content of speech must be necessary to advance a compelling government interest. While conceding that the government has a compelling interest in avoiding a violation of the Establishment Clause, the Court in Widmar held that an equal access policy would not have a purely religious purpose, a primarily religious effect or excessively entangle religion in violation of that clause. Thus, the city ordinance here would be struck down because it discriminates on the basis of the content of speech, and the discrimination does not serve any compelling government interest.

How well did you know this?
1
Not at all
2
3
4
5
Perfectly
43
Q

Easement by necessity beyond maintenance and repair

A

The developer has an implied easement by necessity, since there is no other access to the property. (In this case, there was some use of the road before the properties were divided, so “strict” necessity is not required.) The owner of an implied easement has a limited right to upgrade the easement and generally may not develop or upgrade it beyond the reasonable contemplation of the parties at the time the property was divided. This choice is correct, because the daughter sold the land to the developer knowing that it would be used for townhomes and that the homeowners would need access to the public road. It was thus reasonably within the contemplation of the daughter and the developer that the road would eventually need to be paved.

How well did you know this?
1
Not at all
2
3
4
5
Perfectly
44
Q

Assigning un-assignable contracts

A

As a general rule, all contracts are assignable and delegable except personal service contracts and long-term requirement contracts. Even though there may be an anti-assignment provision in a contract, it does not prevent the assignor from assigning his rights. When an assignor makes an assignment in violation of an anti-assignment clause, the assignment is valid. However, the obligor does have a cause of action against the assignor for breach of contract (but ordinarily, damages will be nominal). Because the contract here specified that delivery was “F.O.B. seller’s place of business,” the risk of loss passed from the seller to the buyer as soon as the seller placed the chairs in the hands of the carrier for shipment. As such, the buyer bore the risk of loss, and the friend will be able to claim the entire contract price from the buyer.

How well did you know this?
1
Not at all
2
3
4
5
Perfectly
45
Q

Defamation of a public official

A

Defamation of a public official requires the plaintiff prove that the defendant acted with actual malice: intentional knowledge of falsity or a reckless disregard of the truth. Negligence–which is what is present in this case–is insufficient as a First Amendment matter to impose liability [New York Times v. Sullivan, 376 U.S. 254 (1964)].

How well did you know this?
1
Not at all
2
3
4
5
Perfectly
46
Q

Miranda violation statements

A

Once Miranda warnings have been given, a defendant who has requested an attorney may not be further questioned until either counsel is furnished or the defendant voluntarily initiates a discussion. Here, given that the man asked to speak with an attorney before he answered the officers’ questions and that he did not voluntarily initiate the subsequent discussion, the officers in fact violated the man’s Miranda rights when they questioned him further about his accomplice. As such, the information the officers obtained would not be admissible to prove that the man had worked with that particular person to commit the burglaries. However, although statements taken in violation of Miranda rights may not be used substantively, they may be used to impeach a testifying defendant. Thus, once the man took the stand and testified that he did not know anything about who had committed the burglaries, he opened the door for the prosecutor to introduce his prior statement for purposes of impeachment.

How well did you know this?
1
Not at all
2
3
4
5
Perfectly
47
Q

Defamation

A

This same defamation issue has been tested on the MBE. It is important to remember that the defamatory meaning of a statement must be communicated. Thus, the utterance must be understood by the third person. In general, words spoken in a foreign language will not be actionable unless they are heard by someone who understands that language and what was said. Because the first woman did not understand the voicemail, which had been left in Spanish, this is the best answer choice.

How well did you know this?
1
Not at all
2
3
4
5
Perfectly
48
Q

Substantial alteration by a mechanic

A

Answer (B) is correct because the accident was the result of the mechanic’s alteration of the product, not the result of negligence by the manufacturer or a defect that was in the product when it was sold. This question mirrors the facts of Pichardo v. C.S. Brown Co. [827 N.Y.S.2d 131 (N.Y. App. Div. 2006)]. The manufacturer of a product can be liable under several theories, including negligence, strict tort liability, and breach of warranties. All of these theories are based on the premise that there was something wrong with the product when the manufacturer produced and sold it. The product may have been designed in a dangerous way, assembled improperly, or lacked adequate warnings. The manufacturer might be liable because it acted negligently, or it may be liable because the product was defective even though the manufacturer acted with reasonable care. But under any of these theories, the manufacturer is liable because there was something about the product that was flawed and dangerous. If there was nothing wrong with the product when it left the manufacturer’s hands, the manufacturer will not be liable merely because some third party later altered the product in a way that was improper and dangerous. Here, the snowblower was safe and worked properly until it was modified by the mechanic. The manufacturer attempted to warn people not to modify the product in that manner. It therefore would not be possible for the injured homeowner to show that the manufacturer was negligent, the product was dangerously defective when sold, or the manufacturer breached a warranty with respect to the product.

How well did you know this?
1
Not at all
2
3
4
5
Perfectly
49
Q

Attractive nuisance

A

For a condition on land to be considered an attractive nuisance, there must be evidence that the landowner has reason to know that children are likely to trespass, as well as evidence that the injured party did not appreciate the risk involved. No such evidence is mentioned in these facts; there is no suggestion that children often stray from the day-care center. The corporation should prevail, because the obviousness of the risk, buttressed by the warning signs, should have been appreciated by a “bright 12-year-old child.”

How well did you know this?
1
Not at all
2
3
4
5
Perfectly
50
Q

Privilege of necessity

A

Answer (C) is correct because under the privilege of necessity, a person in peril is entitled to use another person’s property to avoid a dangerous situation. Necessity means that there may be times when danger makes it justifiable for you to infringe on someone else’s property rights. For example, ordinarily you would be liable for trespassing if you entered another person’s property without permission. But if you ran across the property to escape an angry grizzly bear, you would not be liable for trespass because you acted out of necessity to save your life. If your infringement of the person’s property rights is done out of necessity, that person is not entitled to use force to stop you from using the property to save yourself. The situation in this question is somewhat similar to the famous old case of Ploof v. Putnam [71 A. 188 (Vt. 1908)], where a family was on a boat during a dangerous storm, and they tried to save themselves by mooring the boat to the defendant’s dock. The defendant’s servant drove them away, preventing them from using the dock, and they were injured when the storm destroyed their boat. The court held that the family acted out of necessity, and so the defendant was not entitled to prevent them from using his property and could be held liable for doing so. Likewise, the pilot acted out of necessity by making an emergency landing, so he would not be liable for trespass for landing the plane there, and the homeowner was not entitled to send his dogs to attack the pilot.

How well did you know this?
1
Not at all
2
3
4
5
Perfectly
51
Q

Negligent infliction of emotional distress

A

Answer (D) is correct because the widow could have a claim for negligent infliction of emotional distress. Many years ago, courts generally declined to let people recover on negligence claims where the alleged harm was emotional in nature. If someone had a physical injury, such as a broken leg, they could recover for that, including the emotional harm connected with the physical injury. However, damages for emotional harm alone, without an accompanying physical injury, could not be recovered. Courts eventually began to make special exceptions where they would hold someone liable for negligence causing emotional harm. One of those special exceptions was for situations involving the negligent mishandling of a corpse, so that people could recover if they were emotionally harmed by finding out that the dead body of a beloved family member had been negligently lost, damaged, or destroyed. Most courts eventually went further and created broader exceptions, allowing recovery where the plaintiff experienced some physical impact in the incident that produced the emotional distress, the plaintiff was in a place (a “zone of danger”) where there was a risk of being physically hit by something, or the plaintiff had physical manifestations or symptoms of the emotional distress. Many courts have now gone even further, allowing recovery for negligent infliction of emotional harm without special limitations or requirements. States thus have a variety of different approaches to negligent infliction of emotional distress claims. In most states, it would be possible to recover in the situation described by this question, either under a special exception for cases about negligent mishandling of corpses or under a broader rule that generally allows recovery for negligent infliction of emotional distress without special restrictions or limitations. It is reasonably foreseeable that the hospital’s mistake would cause the widow to suffer serious emotional distress.

How well did you know this?
1
Not at all
2
3
4
5
Perfectly
52
Q

Georgia per stirpes succession

A

Georgia follows a per stirpes intestate distribution scheme in which the decedent’s descendants take by right of representation. The primary division of the estate is at the first generation that has any living members. The right of representation allows the children or issue of an heir who predeceased the decedent to take their parent’s or ancestor’s share before and above anyone else, but the estate of a deceased heir with no descendants of his/her own does not take a portion of the decedent’s estate. In this case, the decedent has no spouse. The generation at which the decedent’s estate will be divided is the decedent’s children. The decedent has two living children and one deceased child who left his own heirs, so the estate will be divided into three equal shares. The living son and daughter each take one-third. The children of the deceased son take the deceased son’s one-third share by their right of representation, each getting an equal one-sixth of the decedent’s estate. Therefore, this answer is correct.

How well did you know this?
1
Not at all
2
3
4
5
Perfectly
53
Q

Georgia will two witness rule

A

Under Georgia law, a will must be written, signed by a competent testator, and signed by two or more competent witnesses. It is not necessary that the subscribing witnesses sign in the presence of each other; it is sufficient if each signs in the presence of the testator. In this case, the facts indicate that the testator signed the will and that both neighbors signed the will in front of the testator. The fact that the two witnesses did not sign in the presence of the other is immaterial. As such, the will was properly attested.

How well did you know this?
1
Not at all
2
3
4
5
Perfectly
54
Q

Georgia properly executed codicil

A

Georgia law requires that a will must be signed by at least two competent witnesses. In this case, the testator did not properly execute his will because only the uncle served as a witness. However, under Georgia law, a validly executed codicil validates an invalid will if the codicil refers to it with sufficient certainty to identify and incorporate it. In this case, the facts indicate that the testator properly executed the codicil and that the codicil specifically incorporated the will. Therefore, the codicil validated the testator’s will, and the uncle may be entitled to inherit under the will.

How well did you know this?
1
Not at all
2
3
4
5
Perfectly
55
Q

Georgia anti-lapse statute

A

At common law, a gift to a beneficiary in a will lapses (does not take effect) if the beneficiary dies before the testator does. However, under Georgia law, if a beneficiary is dead when the will is executed or otherwise dies before the testator, but has any descendants living at the death of the testator, the testamentary gift, if absolute and without remainder or limitation, shall not lapse but shall vest in the descendants of the beneficiary in the same proportions as if inherited directly from the deceased beneficiary under the intestacy laws of this state. Therefore, the GA anti-lapse statute will apply, and the gift to the testator’s niece will pass to the niece’s heirs.

How well did you know this?
1
Not at all
2
3
4
5
Perfectly
56
Q

Will destruction

A

A will may be revoked by physical destruction of the will by the testator or by a third party at the testator’s direction. In this case, the mother did not direct the neighbor to tear up the will and only threatened to disinherit her children. Therefore, the mother did not revoke her will and her assets should not be distributed as though she died intestate. Thus, this answer is correct.

How well did you know this?
1
Not at all
2
3
4
5
Perfectly
57
Q

Marriage after a will

A

When a testator has made a will prior to marriage but does not include in that will a provision in contemplation of marriage, the subsequent spouse will receive the share of the estate that the spouse would have received if the testator had died without a will (intestate). In this case, the man had a will that was executed prior to his marriage. The will did not include a provision in contemplation of the marriage. Therefore, his wife will receive the share she would have received if the man had died without a will. Thus, this answer is correct.

How well did you know this?
1
Not at all
2
3
4
5
Perfectly
58
Q

Seller to cure defect despite perfect tender rule

A

Section 2-601 of the Uniform Commercial Code creates the perfect tender rule, which provides that any goods at the time of delivery that do not conform to the contract may be wholly rejected, wholly accepted, or partially accepted. As such, the driver had the right to reject the car. However, Section 2-508 also allows a seller the right to cure the defect if the goods tendered are rejected at the time of delivery. Pursuant to this provision, the seller can notify the buyer that it intends to cure the defect and is permitted a reasonable period of time to do so. Here, because there was no stated deadline for the delivery of the car, the dealership is permitted a reasonable time to cure the problem with the car’s seats. As such, this answer best describes the parties’ rights as of December 14.

How well did you know this?
1
Not at all
2
3
4
5
Perfectly
59
Q

Rejecting nonconforming goods

A

According to UCC Section 2-602, notice of intent to reject nonconforming goods must be communicated within a reasonable time from their delivery. The store owner failed to inform the manufacturer that she was rejecting the shipment for well over three months, so the manufacturer is entitled to the full contract price.

How well did you know this?
1
Not at all
2
3
4
5
Perfectly
60
Q

Nonmutual issue preclusion

A

Issue preclusion prevents relitigation of issues that were fully and fairly litigated and were necessarily decided in a proceeding that reached a final judgment on the merits. Nonmutual issue preclusion is when a party seeks to prevent litigation of an issue that the other party has unsuccessfully litigated before a different party. If the two claims were brought in two different jurisdictions, the issue of what law to apply arises. If the first case was decided in a state court, the second court will apply the preclusion rules of the first court. This is true whether the second court is a state or federal court. Consequently, because the first court applied the rules of State A, the second court will not allow nonmutual issue preclusion.

How well did you know this?
1
Not at all
2
3
4
5
Perfectly
61
Q

Stipulated jury size

A

Generally, “a jury must begin with at least six and no more than 12 members” [Fed. R. Civ. P. 48(a)]. However, parties may stipulate to a verdict returned by less than six jurors [Fed. R. Civ. P. 48(b)]. Here, because the parties stipulated to a verdict from a five-person jury, the court should not grant the man’s motion.

How well did you know this?
1
Not at all
2
3
4
5
Perfectly
62
Q

Judge findings

A

In an action tried on the facts without a jury or with an advisory jury, the judge is required to find the facts specially and state his or her conclusions of law separately. The findings and conclusions may be stated on the record after the close of evidence or may appear in an opinion or a memorandum of decision filed by the court [Fed. R. Civ. P. 52(a)(1)]. Without such a statement, an appellate court would not be able to determine why the judge ruled in favor of the manufacturer, and so reversal on these grounds would be appropriate [United States v. Forness, 125 F.2d 928 (1942)].

How well did you know this?
1
Not at all
2
3
4
5
Perfectly
63
Q

Purposeful availment

A

For a federal court to assert personal jurisdiction under a minimum contacts analysis, the defendant must have established a minimum contact with the forum state, the claim against the defendant must be related to that contact, and the exercise of jurisdiction must not offend traditional notions of fair play and substantial justice. To establish a minimum contact, the defendant must purposefully avail itself to the privilege of conducting activities within the forum state [World-Wide Volkswagen Corp. v. Woodson, 444 U.S. 286 (1980)]. Here, only 2% of the subsidiary’s tires are distributed in State A by the parent corporation, not the subsidiary. Consequently, the subsidiary has not purposefully availed itself of the privilege of conducting activities within the forum state since it did not actually distribute its tires there.

How well did you know this?
1
Not at all
2
3
4
5
Perfectly
64
Q

Federal question

A

For federal-question jurisdiction to exist, the underlying claim for relief must “arise under” federal law. 28 U.S.C. Sec. 1331. Federal-question jurisdiction does not exist merely because federal law is cited solely for the purpose of calculating damages. Federal law is not the law that creates the ranchers’ tort claim, and therefore federal-question jurisdiction does not exist

How well did you know this?
1
Not at all
2
3
4
5
Perfectly
65
Q

No evidence at the summary judgment stage

A

There is no requirement that an opposing party file evidence in opposition to a summary-judgment motion. First Nat. Bank of Arizona v. Cities Serv. Co., 88 S.Ct. 1575 (1968). The party can simply argue that the moving party’s evidence is insufficient to warrant judgment as a matter of law. 10A Wright, Miller & Kane, Federal Practice & Procedure Sec. 2727.2. Therefore, the failure to file evidence is not itself a basis to grant summary judgment.

How well did you know this?
1
Not at all
2
3
4
5
Perfectly
66
Q

Bringing leaned treatises into evidence

A

Federal Rule of Evidence 803(18), the learned treatise hearsay exception, provides that if the court finds a publication to be a reliable authority, then “statements” from it may be read into evidence, but the publication may not be received as an exhibit. Thus, the jury is not allowed to bring learned treatises into the jury room. There is a concern that if juries were allowed unrestricted access to the whole publication, they might rely on parts of the publication that are not germane to the case. Moreover, the intent of the rule is that juries need to be guided through the pertinent parts of the publication by the testifying experts.

How well did you know this?
1
Not at all
2
3
4
5
Perfectly
67
Q

Default judgment by the clerk or court

A

A clerk may enter default judgment only when the amount sought is a sum certain and the defaulting party has failed to appear. Fed. R. Civ. P. 55(b)(1). Here, because the defaulting party–the builder–is the original plaintiff in the action and thus appeared when filing the complaint, the clerk is precluded from entering a default judgment.

How well did you know this?
1
Not at all
2
3
4
5
Perfectly
68
Q

Military reassignment for religious reasons

A

This correct answer choice states the standard set by the Supreme Court in Employment Division v. Smith [494 U.S. 872 (1990)]. Although the 20-year-old was seeking reassignment for religious reasons, the regulation requiring waiver of educational benefits was one of general applicability and applied regardless of religion. In light of the Smith case, the 20-year-old most likely will not be able to refuse to sign the waiver of educational benefits if he wants civilian reassignment–at least not based on violation of the Free Exercise Clause. A different outcome might occur if it turned out that the only grounds for civilian reassignment are religious, but the facts don’t suggest it. Furthermore, although the government cannot determine the truth or falsity of a person’s religion, they can determine that person’s sincerity with regard to his religious beliefs. If the military investigates the 20-year-old’s membership in the church for this purpose, it is not a violation of the Free Exercise Clause.

How well did you know this?
1
Not at all
2
3
4
5
Perfectly
69
Q

Assignment

A

An assignment arises when the holder of a right, an obligee, manifests the intent to make a present transfer of that right to another, the assignee. Upon an assignment, the assignor’s rights are extinguished and transferred to the assignee. An assignment is to be distinguished from a promise to do something in the future, such as the payment of money. Here, the writing in which the builder promised to pay the lender the $10,000 he received from the homeowner did not transfer to the lender the right to receive payment directly from the homeowner, and thus it did not create an assignment.

How well did you know this?
1
Not at all
2
3
4
5
Perfectly
70
Q

Interference with prospective advantage

A

The tort of interference with prospective advantage protects the probable “expectancy” interests of future contractual relations of a party, such as the prospect of obtaining employment or the opportunity to obtain customers. In the absence of prohibition by statute, illegitimate means, or other unlawful conduct, a defendant seeking to increase his own business may cut rates or prices, allow discounts, or enter into secret negotiations behind the plaintiff’s back, refuse to deal with the plaintiff, or threaten to discharge employees who do. But, where the defendant’s conduct is illegal or malevolent, he may be liable for interference with prospective advantage. Here, the businessman was involved in unlawful conduct because he intimidated the new store’s customers by threatening to hurt them if they attend the new store. Hence, he interfered with the new store’s opportunity to obtain customers and will be held liable.

How well did you know this?
1
Not at all
2
3
4
5
Perfectly
71
Q

injurious falsehood

A

An injurious falsehood is a false statement made to another by the defendant that causes economic injury to the plaintiff. In this case, the businessman did not make any false statement, but was in fact issuing a threat. As such, he was employing intimidation and will be liable for interference with prospective advantage rather than injurious falsehood.

How well did you know this?
1
Not at all
2
3
4
5
Perfectly
72
Q

violation of statute as negligence per se

A

Violation of a statute is negligence per se only if the person harmed is one of the class intended to be protected, if the violation is not excused, and if the risk threatened is the type the statute was intended to protect against. The equipment certification statute is clearly intended to guard against injuries caused by defective machinery. Here, the crane was in perfect working order. Even if it had been certified, the operator’s negligence would still have injured the man. Under these circumstances, it is unlikely that the failure to obtain a certificate would be regarded as negligence per se such that it would conclusively establish the company’s vicarious liability.

How well did you know this?
1
Not at all
2
3
4
5
Perfectly
73
Q

Liability over a newsstand owner

A

A person who distributes the original defamatory message as a commodity (e.g., a bookseller, newspaper vendor, retailer, etc.) is liable only if he knew or should have known that the material distributed contained the defamatory message. Here, the newsstand owner read the paper and knew that the article included the defamatory statement. He also knew that the quarterback did not use steroids. As such, he may be liable as a secondary publisher of a defamatory statement, because he knew that the statement was not true but disseminated it anyway.

How well did you know this?
1
Not at all
2
3
4
5
Perfectly
74
Q

iied

A

It is likely that the lawyer will be found to have committed intentional infliction of emotional distress. The tort of intentional infliction of emotional distress requires that the defendant commit an intentional act amounting to extreme and outrageous conduct that causes the plaintiff severe mental distress. To be liable for this tort, the defendant must intend to cause the plaintiff severe mental distress. It has been argued that recovery should likewise be allowed where the defendant acts “recklessly,” that is, in deliberate disregard of a high degree of probability that the emotional distress will follow. Given that the wife screamed at the lawyer to stop and he refused, the lawyer was aware of the wife’s presence. Even after that point, he continued to hit the economist with the bat. Additionally, the wife was so upset by the incident that she could not bring herself to continue gardening, a hobby that she formerly had loved. That would be considered severe mental distress. Therefore, the lawyer will be found to have intended to cause her emotional distress or to have deliberately disregarded the likelihood that his act would cause her such distress. Battery is an intentional act that causes the plaintiff a harmful or offensive contact. While the defendant is permitted to use reasonable force to prevent the plaintiff from committing a tort against the defendant’s property, the amount of force used by the defendant must be no greater than necessary to prevent the threatened harm. Here, the lawyer hit the economist repeatedly with a bat, long after the economist had ceased throwing rocks at the lawyer’s house. As such, the lawyer cannot assert defense of property as a defense against a claim of battery, and it is true that he is liable for this tort. As such, the lawyer will be held liable for both intentional infliction of emotional distress and battery.

How well did you know this?
1
Not at all
2
3
4
5
Perfectly
75
Q

Public nuisance

A

A private person can file a claim for public nuisance if he sustained a harm different from that of the community at large. If the homeowners can show that they suffered a harm different from that of the community at large, they can prevail against the company under a public nuisance theory.

How well did you know this?
1
Not at all
2
3
4
5
Perfectly
76
Q

Not complying with the motion to compel discovery

A

If a party does not comply with a motion to compel discovery, the requesting party can immediately seek: (1) a court order declaring that the facts sought in the deposition are established in favor of the requesting party; (2) a court order prohibiting the disobedient party from presenting certain claims or defenses; (3) a stay or dismissal of entire action; or (4) an order of contempt. A permissive counterclaim seeking liquidated damages is not a valid remedy for this situation.

How well did you know this?
1
Not at all
2
3
4
5
Perfectly
77
Q

Fundamental right to travel

A

Answer (D) is correct because the law burdens the right to travel. One of the key purposes of the Constitution was to ensure that the states would be truly united as one nation. The Supreme Court therefore has held that the Constitution provides a fundamental right to travel within the United States, and that this includes a right to move to another state, make a home there, and be treated the same as those who have lived in the state for a longer period of time. Strict scrutiny thus applies when a state discriminates against new residents of the state. For example, in Saenz v. Roe [526 U.S. 489 (1999)], the Supreme Court struck down a state law that limited the amount of welfare benefits a person could receive in the first year of residence in the state. Likewise, a law providing less generous health care benefits to new residents of a state would burden the fundamental right to travel and would be struck down unless the state showed a compelling justification for discriminating against its new residents.

78
Q

Embezzlement

A

When the customer gave the clerk the $5 bill to pay for the milk, the clerk gained lawful possession of the $5 bill to deposit in the cash register for his employer. When the clerk put the money directly into his pocket, a conversion occurred. Therefore, the clerk is guilty of embezzlement, defined as the fraudulent conversion of the personal property of another by one in lawful possession.

79
Q

Agency and felony murder and shield exception

A

There are two schools of thought regarding causation and the felony murder rule. The majority view is known as the agency theory of felony murder. Under the agency view, the killing must be caused by one of the felons engaged in the underlying felony. If the killing is caused by a non-felon (e.g., a bystander, victim, or police officer), the felony murder rule generally does not apply. The major exception under the agency theory is the “shield exception.” If one of the felons takes a person as a hostage or shield and that person is killed, all of the participating felons are liable. The minority view is called the proximate cause theory. Under this view, all participating felons are liable for any death that is proximately caused by acts in furtherance of the felony (regardless of whether the shooter is a felon or a third person, such as a bystander), and if the victim’s death is a natural and probable consequence of the underlying felony, all participating felons are liable. This question involves the agency theory of felony murder, which requires that the killing be caused by one of the participating felons, unless the shield exception applies. Under the facts presented, the bank manager was taken hostage by the friend (one of the felons), and was subsequently killed by a police officer. The result is that the defendant is liable for the bank manager’s death under the shield exception. In contrast, the friend was killed by a police officer. Under the agency theory, the defendant is not liable for his death.

80
Q

Assignment and Dumpor case

A

Under the Rule in Dumpor’s Case, a covenant against assignment, once waived, is unenforceable as to subsequent assignments. However, the landlord can reserve the right to restrict future assignments by expressly stating that the waiver is a one-time occurrence. Because the landlord expressly stated that she was agreeing to a waiver “this one time only,” the covenant against assignments will prevent the assignment to the athlete. Therefore, the visiting professor remains liable on the lease. However, the landlord could choose to sue the athlete, thereby ratifying the assignment. If she chose to do so, the visiting professor would no longer be in privity of estate with the landlord, but the athlete would.

81
Q

Free to leave standard

A

The key issue is whether the defendant was in custody at the police station during the questioning. If so, then the defendant should have been given his Miranda warnings, and the first statement would be inadmissible. In determining the custody issue, the Supreme Court today follows an objective approach. Account must be taken of the totality of circumstances: (1) when and where did the interrogation occur; (2) how long it lasted; (3) how many police officers were present; and (4) what the officers and the defendant said and did. First, the defendant voluntarily came to the police station for questioning. In Oregon v. Mathiason [429 U.S. 492 (1977)], it was held that there is no custody when the person comes to the station in response to an “invitation” from the police. The relevant inquiry is not on the policeman’s state of mind; rather whether a reasonable person would have believed that he was free to leave. The fact that the detective told the defendant that he “was free to leave at any time” would seem to outweigh the officer’s unarticulated plan to keep him in the room. As such the defendant’s statement was voluntary and not the product of police coercion. Both the statement and confession should be admitted.

82
Q

ucc paying for nonconforming goods

A

Under Section 2-607(1) of the Uniform Commercial Code (UCC), when a buyer accepts goods delivered by the seller, even nonconforming ones, he must pay at the contract rate for any goods accepted. (Note that the UCC rule providing that a nonconforming shipment is both the acceptance of the offer and simultaneously a breach is not applicable in this case, because the contract had already been formed prior to the seller’s shipment of the Model-Z widgets.) The seller’s act of “accommodation” has no effect on his contractual obligation to deliver Model-Y widgets, unless it resulted in a mutual modification of the contract under UCC Section 2-209, but the facts are not sufficient to show a modification of the contract here. Furthermore, though the buyer would have the right to sue for any damages caused to him by the non-conformity of the goods, there probably are no such damages here because the Model-Z widgets were superior to the Model-Y widgets he had ordered. Therefore, the buyer is liable for the contract price for the goods.

83
Q

Reformation and drafting error

A

The facts here demonstrate an example where one party is likely entitled to a reformation of the contract, and the parol evidence rule is inapplicable where a party to a written agreement alleges facts entitling him to reformation of the agreement. Reformation permits the court to exercise equitable powers to “reform” (rewrite to correct the mistake) a contract due to a drafting error resulting in a failure to capture the true intent of the parties. To obtain reformation, a party must show that: (1) there was an antecedent valid agreement; (2) that is incorrectly reflected in the writing; and (3) proof of these elements is established by clear and convincing evidence. Here, both parties (as well as the notary) knew that the true and full expression of the settlement agreement included a term giving the seller a percentage of the mineral profit rights. However, due to an oversight by the notary while drafting the written form of the agreement, the term was omitted. The fact that no one recognized the omission was a mistake, the result of which was that the provision was omitted from the instrument intended to be the expression of the actual agreement of the parties. Therefore, the judge will admit evidence regarding the antecedent agreement and negotiations, because the parol evidence rule is inapplicable where the allegations go to reformation.

84
Q

suitable nonconformance

A

UCC Section 2-601 provides that if the goods or the tender of delivery fail in any respect to conform to the contract, the buyer may (1) reject the whole; or (2) accept the whole; or (3) accept any commercial unit or units and reject the rest. This is known as the perfect tender rule and under it, the buyer has the right to reject the entire shipment and bring suit for breach of contract. However, UCC Section 2-508(2) gives the seller a chance to cure even if the time for performance has passed. When the time for performance has already arrived (as it had in this question), the seller may rely on Section 2-508(2) if the “…seller had reasonable grounds to believe (the tender) would be acceptable…the seller may, if he seasonably notifies the buyer, have a further reasonable time to substitute a conforming tender.” Although the contract called for the woodchucks to be dressed in fire-engine red sweaters, the seller had reasonable grounds to believe that cherry-red sweaters would have been acceptable because both colors were similar enough that other customers used them interchangeably. Because the toymaker notified the buyer of his willingness to cure and send the proper toys, this late tender provision, Section 2-508(2), will give the seller a reasonable time to cure the nonconformance even though the date for performance has already passed.

85
Q

Contracts Clause

A

The Contract Clause prohibits states from enacting retroactive laws that impair an existing contractual obligation. The primary intent behind the drafting of the clause was to prohibit states from adopting laws that would interfere with the contractual arrangements between private citizens. In determining whether a state law affecting an individual’s ability to carry out obligations under contract is an improper impairment of contract, the U.S. Supreme Court articulated a three-step test: (1) whether the state law has “operated as a substantial impairment of a contractual relationship”; and, if so, (2) whether the state law is designed to promote a significant and legitimate public purpose; and, if it does, then (3) whether the law adjusting the contract obligations was a reasonable and narrowly tailored means of promoting the significant public purpose identified in step two rather than an unjustifiable attempt to merely change the obligation of parties to a private contract. Here, the statute does not promote a significant and legitimate public purpose. Therefore, the requirements contract is still enforceable.

86
Q

Superior skill for negligence

A

In determining the standard of conduct for negligence, if a person has knowledge, skill, or even intelligence superior to that of the ordinary person, the law will demand of that person conduct consistent with it. Experienced milk haulers, hockey coaches, expert skiers, construction inspectors, and doctors must all use care that is reasonable in light of their superior learning and experience, and any special skills, knowledge, or training they may personally have over and above what is normally possessed by people in the field. Certainly, a professional racecar driver will be required to utilize his superior driving skills in the operation of his motor vehicle. Therefore, this answer choice is correct.

87
Q

Deed absolute as mortgage

A

It has long been recognized in equity that a deed absolute intended for security will, in fact, be construed as a mortgage. This is not really surprising when it is remembered that the traditional form of a mortgage was a conveyance subject to defeasance, and that the equity of redemption was created by the equity court to protect the mortgagor after default. In order to preserve this equity of redemption, various rules were formulated to prevent mortgagees from limiting or clogging the equity of redemption. The most common example of such rules is the principle “once a mortgage always a mortgage.” This, in effect, means that a mortgagee cannot circumscribe the mortgagor’s right to redeem by disguising the transaction as an outright conveyance. The facts state that the deed absolute was intended as additional security. As a consequence, the deed will not extinguish the debtor’s right of redemption.

88
Q

Continuation of jeopardy

A

Double jeopardy–being tried twice for the same crime by the same sovereign–can be an issue in retrials after appeals and after mistrials. The issue here is that a retrial following a defendant’s successful appeal is considered to be a continuation of the earlier proceeding. There is only one “jeopardy” as a result.

89
Q

positive and unequivocal repudiation

A

Certainly, if one party to an executory bilateral contract repudiates the contract in advance of the time set for performance by announcing that (s)he will not perform it, the other party may treat such anticipatory repudiation as a present total breach. Students should be aware, however, that (for an action to be for anticipatory repudiation) the repudiation must be positive and unequivocal. An expression as to future inability to perform is not breach by repudiation. In the present fact situation, the contractor merely indicated “he didn’t think he could perform the remodeling work.” That in itself is not a positive and unequivocal repudiation; therefore, the owner is in breach of contract because he refused to permit the contractor to perform the remodeling on the performance date. Where the owner has reasonable grounds to believe the contractor has manifested a prospective inability to perform, he may demand adequate assurances of performance. If assurances are not forthcoming within a reasonable time, he may treat the situation as a repudiation. In this question, however, the owner never notified the contractor about his concerns. Therefore, the contractor was justified in showing up ready to begin work on May 1. The owner’s action in hiring the carpenter was a breach of the owner-contractor agreement. The contractor will prevail.

90
Q

Joint participant exception

A

This question addresses an exception to the marital communications privilege called the joint participants exception. In federal courts and in most states, there are two recognized marital privileges. First, the spousal testimonial privilege, which allows a witness-spouse to refuse to testify against his/her spouse in a criminal (but not civil) proceeding. However, the spouse-witness may voluntarily testify against his/her spouse as to nonconfidential communications made during the marriage [Trammel v. United States, 445 U.S. 40 (1980)]. The testimony privilege only applies while the spouses are presently married to each other, but it extends to pretrial matters as well. Former spouses may not assert the testimony privilege or prevent former spouses from testifying. The second privilege is the marital communications privilege, which protects confidential communications made during the existence of a valid marriage intended by the spouses to remain confidential. The communications privilege applies in both criminal and civil proceedings, and either spouse may invoke the privilege. However, there are three recognized exceptions to the communications privilege: (1) suits between the spouses (including divorce and child custody disputes); (2) suits where one spouse is charged with a crime or tort against the other or the children in the household; and (3) where the spouses jointly participate in criminal acts–i.e., the joint participant exception. Thus, this answer choice states the correct result under the joint participants exception; neither privilege applies when both spouses actively participate in an ongoing crime.

91
Q

dog on a leash is the extension of a person

A

Torts is tricky, because questions often present a generic call of the question, forcing you to keep open the realm of possible areas. The tort action could be an intentional tort, an action in negligence, or an action in strict liability. Here, you are forced to decide between negligence and intentional torts. This is the correct answer choice, not because the man might have violated a statute by entering on the “Do Not Walk” signal, or even because the pedestrian complied with one by having the dog leashed. The pedestrian will prevail because the dog was on a leash. Normally, a battery requires the offensive touching of the person of another. However, there can also be a battery if the defendant comes in contact with something connected to the plaintiff. Here, the dog on a leash was an extension of the pedestrian’s person, and so once the man kicked the dog, he committed a battery against the pedestrian.

92
Q

Abuse of discretion and expert testimony

A

Under the Frye standard, the admissibility of scientific expert testimony was based on whether it had “gained general acceptance in the particular field in which it belongs” [Frye v. United States, 293 F. 1013 (D.C. Cir. 1923)]. The Supreme Court in Daubert v. Merrell Dow Pharmaceuticals, Inc. [509 U.S. 579 (1993)] rejected the Frye “general acceptance” standard. The Court in Daubert held the task of “gatekeeping” scientific expert testimony (i.e., whether that testimony is from scientific knowledge) rests on the trial judge. In General Electric Co. v. Joiner [522 U.S. 136 (1997)], the Court ruled that the appropriate appellate standard for admitting or excluding scientific expert testimony is an “abuse of discretion standard” in applying the Daubert “gatekeeping” rule of trial courts under Rule 702. In Joiner, the Court held that in applying the abuse of discretion standard regarding the admissibility of scientific testimony, an appellate court “may not categorically distinguish between rulings allowing expert testimony and rulings which disallow it.” As such, following the Joiner decision, this is the correct answer.

93
Q

Prisoner segregation

A

The issue of racial segregation of inmates has twice been addressed by the U.S. Supreme Court. First, in Lee v. Washington [390 U.S. 333 (1968) (per curium)], the Supreme Court unanimously upheld a three-judge district court opinion declaring that racial segregation in the Alabama prison system violated the Equal Protection Clause. More recently, in Johnson v. California [543 U.S. 499 (2005)], the Supreme Court, in a 5-3 opinion, ruled that “all racial classifications imposed by the government must be analyzed under strict scrutiny,” and that a prison policy of even temporary racial segregation must be “narrowly tailored to serve a compelling state interest.”

94
Q

Excavation and collapsing adjacent land

A

A landowner may be strictly liable if his excavation causes adjacent land to subside (sink). If the adjacent land is improved, strict liability applies only if the adjacent land would have collapsed in its natural state. Even if the adjacent land would not have collapsed in its natural state, the landowner is liable for damages done negligently and this liability extends to structures on the land. Because the facts state that the butcher proved that his land would have subsided anyway even without the shop on the land, then the developer would be strictly liable for the damage caused by the subsidence, including damages to the butcher’s building. Therefore, this answer choice is correct.

95
Q

Boundary line by acceptance

A

Boundary line questions frequently appear on the bar exam both on the essays and on the MCQ exam. It often happens that an owner is uncertain as to the precise location on the ground of the parcel’s boundaries. This uncertainty may result from an ambiguous or confusing description in the deed or other documents, but it is not always easy to locate boundaries even if the written description is clear. In such cases, abutting owners frequently enter into informal agreements that fix the boundaries between them. In general, the law tends to uphold these determinations, even though they often have the effect of placing a boundary somewhere other than at its “true” location. To establish a boundary by agreement, it must first be shown either that the parties were uncertain or unaware of the correct location, or that a dispute as to the true location existed between them. In establishing a boundary by agreement, most courts also require the taking (and relinquishing) of possession by the parties as to the agreed line. Possession serves an evidentiary purpose here, and is roughly analogous to the part performance doctrine in land sale contracts. The boundary line agreement between the man and the woman allowing construction of the fence was valid.

96
Q

oath or affirmation required to testify

A

Federal Rule of Evidence 603 states that “before testifying, every witness shall be required to declare that the witness will testify truthfully, by oath or affirmation….” Rule 603 is a mandatory rule leaving no discretion to the court. Under Rule 603, a witness who refuses oath or affirmation may not testify.

97
Q

materiality and implied consent

A

When considering the issue of medical malpractice, one area of litigation involves the doctrine of implied consent. The question sometimes arises in medical or surgical treatment cases as to whether the doctor is aware that the patient does not understand the nature of the operation or the risks of undesirable consequences involved in it. Under the modern theory of liability, the focus is on the standard of professional conduct. In deciding whether the failure to disclose certain risks does, in fact, fall below the standard of professional conduct, courts will often inquire into the materiality of the risk. In other words, the physician’s duty is to disclose all risks that are “material.” The ways to tell if the risk was material was to look at whether physicians customarily would inform their patients of the kind of risk in question or whether the physician would have made the disclosure under these circumstances. Another line of more modern cases would define materiality in terms of whether a reasonable person in the plaintiff’s position would have wanted to know about this risk. This is the best choice since it presumes that the operation was performed successfully and there were no damages. Thus, the doctor would be more certain to prevail under this reason.

98
Q

instructions are nonassertive

A

The doctor’s instructions to the boy’s mother would be considered verbal conduct, which falls within the definition of a statement according to Federal Rule of Evidence 801(a)(2) as an implied assertion. However, instructions are generally considered nonassertive verbal conduct, because they are used to prove an inference, not establish the truth of the matter being asserted–in this case, it infers that the boy is feeling pain, not that the mother was giving the boy the pills according to the instructions. Therefore, while the instructions are a statement, they would not be considered hearsay, and so the mother is free to give this testimony because she had firsthand knowledge of what was said by the doctor. As such, the motion should be denied.

99
Q

Search inside the car

A

The stop was lawful due to the broken vent window. After running the driver’s information, the officer was aware that the driver had an outstanding warrant for his arrest. This provided probable cause for arrest. Therefore, a search incident to arrest would allow for more than a limited pat-down. However, in order for the interior passenger compartment to be searched, there must either be a concern for officer safety or the opportunity for fruits of the arrest to be found within the passenger compartment. Neither are indicated by the facts presented here. Because the probable cause concerns the driver, and not the vehicle, a search of the trunk would be impermissible. Although it is likely that the driver’s car will be impounded, an inventory search would not be proper on the scene either.
A search of the driver, the interior passenger compartment of his vehicle, and also the trunk. It is true that a search of the driver is permissible, as a warrantless search incident to arrest. However, in order for the interior passenger compartment to be searched, there must either be a concern for officer safety or the opportunity for fruits of the arrest to be found within the passenger compartment. Neither are indicated by the facts presented here. Because the probable cause concerns the driver, and not the vehicle, a search of the trunk would be impermissible. Although it is likely that the driver’s car will be impounded, an inventory search would not be proper on the scene either.

100
Q

no cleric led prayer

A

As a general rule, religious activities conducted in public schools violate the Establishment Clause because their primary purpose is to promote religion. The Supreme Court found a public school’s inclusion of a cleric-led nondenominational prayer in its graduation ceremony to violate the Establishment Clause [Lee v. Weisman, 505 U.S. 577 (1992)]. This is so where a state official arranges for the prayer, and the state, as a practical matter, compels attendance and participation in the ceremony. A state may not place students who object to such an exercise in the dilemma of participating in the exercise or protesting against it.

101
Q

email to 3p for sof

A

A signed email from a buyer to her friend describing a deal the buyer recently entered into with a seller for a parcel of land. The seller is now suing the buyer for specific performance. An email from a party to the transaction to a third party describing the agreement will satisfy the writing requirement of the Statute of Frauds. As such, this answer choice is incorrect.

102
Q

destruction of newly constructed building vs existing building

A

Allocation of risk in the building-contract field is a heavily tested area on the multistate portion of the bar exam. There are two distinct situations that need to be distinguished. First, there is the case (as in the present example) when a partially constructed new building or structure is destroyed by a fire or act of God without fault of either party. In this situation, the risk should be allocated to the builder. In contrast, where an existing structure is destroyed by fire or act of God, without the fault of either party, the risk will fall on the property owner. On the bar exam, students will be required to distinguish between the destruction of a newly constructed building as opposed to the destruction of an existing structure. Because this question deals with a newly constructed building, the builder will bear the risk. As a result, the owner will be entitled to recover the $90,000 he has paid to the builder, plus an additional $10,000, which represents the additional cost to complete.
This question deals with allocating risk of loss in construction contracts. As a starting point, the general rule states that a builder who promises to erect a building is still bound to do so, even though the nearly completed structure is destroyed by forces beyond the builder’s control. Furthermore, the builder is liable not only for progress payments received, but also for any damages the owner can prove for loss of expectation. However, the result is different where the builder contracts to do repair or renovation on an existing building. In this case, the existence of the building is regarded as an implied condition to the parties’ agreement, and its destruction, without fault of either party, will discharges the builder’s duty. To summarize, the builder bears risk of loss for fortuitous casualty during new construction, whereas the builder’s duty is discharged where unanticipated difficulty causes destruction during repairs or renovation of an existing structure. In this question, the builder was renovating the beach home by constructing the redwood deck. The home’s destruction during the hurricane discharged the builder’s duty to complete the deck. Where contract duties are discharged in this situation, the builder is entitled to restitution for any benefit that he has conferred on the other party by way of part performance or reliance. This is, therefore, the best answer.

103
Q

No removal after 1 year

A

A case removable on the basis of diversity jurisdiction may not be removed more than one year after the action was commenced [28 U.S.C. Sec. 1446]. Here, the claim could be removed based on complete diversity of citizenship and an amount in controversy over $75,000, but over one year has passed since the citizen filed her claim.

104
Q

Larceny by false pretenses

A

This choice is incorrect because larceny by false pretenses occurs when the defendant, with the intent to steal the property, makes a false statement of a past or present material fact that causes the rightful possessor of the property to turn over title to the defendant. Here the neighbor was only turning over possession of the bicycle for the purpose of a test-ride.

105
Q

Larceny by trick

A

Larceny by trick occurs when the defendant, with the intent to steal the property, makes a false statement of a past or present material fact that causes the rightful possessor of the property to turn over its possession to the defendant. Here, the defendant intended to steal the bicycle, and falsely told the neighbor that he only wanted to test-ride the bicycle. This caused the owner to give possession of it to the defendant. The defendant then kept the bicycle.

106
Q

Consumer expectation test standard

A

The court will use the consumer expectation test standard. The consumer expectation test standard is used by courts in products liability cases involving products with design defects. Under the consumer expectation test standard, a product is in a defective condition unreasonably dangerous when it is more dangerous than would be contemplated by the ordinary consumer who purchases it, with the ordinary knowledge common to the community as to its characteristics. However, this case involves a product with a manufacturing flaw, not a design defect. Therefore, this standard is inapplicable.

107
Q

Danger-utility test

A

The court will use the danger-utility test approach. The danger-utility test approach is used by courts in products liability cases involving products with design defects. Under the danger-utility test approach, a product is defective if a jury determines that the danger it threatens (the cost in human injury and property damage) outweighs its utility to society. However, this case involves a product with a manufacturing flaw, not a design defect. Therefore, this standard is inapplicable.

108
Q

Intrusion into seclusion

A

One type of invasion of privacy is intrusion into seclusion. That appears to have happened here, as the homeowner’s intrusive conduct would be highly objectionable to a reasonable person. Therefore, this is the best answer.

109
Q

Assignment of an output contract

A

Since the farmer’s services were not personal, he could delegate those duties to another without breaching the contract with the businessman. When the farmer sold his property to his daughter, the daughter (as delegatee) could then discharge the duties of the farmer (the obligor) to the businessman (the obligee) by selling the businessman the peaches. However, if the daughter fails to perform satisfactorily, the businessman retains all rights against the farmer, unless there has been a novation. There are no facts here to suggest a novation.

110
Q

Intended to frighten is battery

A

The landowner intended to place anyone walking along the footpath in reasonable apprehension of an imminent harmful contact. This is sufficient intent to support liability for assault. Generally, if a defendant intends to commit assault, battery, false imprisonment, trespass to land or to chattels, and inadvertently commits a different one of these other torts, his intent will “transfer” to support the tort appropriate for the result achieved. Therefore, because of transferred intent, the landowner is considered to have intentionally caused a harmful contact, which makes the landowner liable for battery.

111
Q

disporportionate excess demand on requirements contract

A

In their prior course of dealings, the zoo had always purchased 500 pounds of fish food per month. Since the order in question was disproportionate to the prior practices in which the zoo and the supplier had engaged over the last 10 years, the zoo was not obligated to buy its excess requirement from the fish-food supplier. Therefore, this answer is correct.

112
Q

No intent element on appropriation of identity or likeness

A

One who uses without authorization the name or likeness of another for commercial advantage (to promote a product or service) is subject to liability for the invasion of privacy tort of appropriation of identity or likeness. There is no intent element to this tort, so it is irrelevant that the pharmacist believed that the supermodel’s photograph in the display was authorized. Here, all of the elements of appropriation of identity are present, so the supermodel will prevail in her claim.

113
Q

Public policy consideration in assumption of risk

A

An express assumption of risk in a disclaimer clause is valid only if three criteria are met: (1) the plaintiff is aware of its terms; (2) the injury which occurs is within the risks of which the plaintiff agreed to relieve the defendant; and (3) the disclaimer is not contrary to public policy. The document the young man signed will not be interpreted as an express assumption of risk barring his recovery because the disclaimer is contrary to public policy. A disclaimer is contrary to public policy when it is actually involuntary because the service is a critical need and the plaintiff has no other effective option. The young man needs to get to the hospital for treatment of his back injury. If the ambulance service is the only one available, the young man has signed the disclaimer under force of necessity and it will probably not be enforced against him.

114
Q

Unilateral contract or offer to form bilateral contract

A

The key to this problem is to be able to characterize the letter as an offer to form a bilateral contract, rather than a unilateral contract. If the offer proposed a bilateral contract, the only way to effectively accept it would be by a return promise. On the other hand, if the offer proposed a unilateral contract, the only way to effectively accept it would be by rendering the performance requested. Although the language in the letter does not expressly request a return promise, the majority of courts would interpret it as a bilateral contract. The sentence in the letter regarding the August 9 arrival date did not mean that the camp owner was requesting acceptance by performance, as in a unilateral contract; rather, the arrival date was one of the express terms of the offer. In addition, even if the wording of the offer could be understood as inviting acceptance by either a return promise or by performance, under the traditional and majority view, courts would treat the offer as proposing a bilateral contract. Under the circumstances, the camp owner would need to know if a clown was going to arrive and perform. It would be unreasonable to expect a performer just to show up at camp ready to perform; therefore, courts would construe this offer as requiring a promise in response. Therefore, the only way for the first clown, the offeree, to effectively accept the offer was by communicating a return promise, which she did not do. The clown’s attempted performance by showing up at the camp on August 9 did not constitute a valid acceptance, so the camp owner still had the power to revoke her offer. [By contrast, under UCC Section 2-206, an offer to buy goods may be accepted by communicating a return promise (as in a common law bilateral contract) or by performance–i.e., shipment of the goods (as in a common law unilateral contract). This UCC provision effectively abolishes the common law distinction between bilateral and unilateral contracts for contracts involving the shipment of goods.]

115
Q

Deposing a corporation

A

A corporation may be deposed. The corporation designates one or more persons whose answers will bind the corporation. With respect to any deposition where the deponent is a party, the deposition is scheduled by serving a notice of deposition on all of the lawyers. Here, the proposed deponent is the defendant, a corporation. As such, it can be deposed if the plaintiff serves a notice of deposition on all counsel.

116
Q

Failure to prosecute when not complying with court orders

A

Simply, if a plaintiff fails to comply with a federal court order, the plaintiff puts itself in danger of dismissal for failure to prosecute, loss of any associated request, and/or sanctions, depending on the circumstances of the case. Here, although the plaintiff had engaged in settlement negotiations with the defendant, it did not involve the court in those negotiations or follow the deadlines and orders put forth by the court.

117
Q

Corporate citizenship and right of removal

A

The general rule under 28 U.S.C. Section 1441(a) is that a corporation may remove a case filed in state court to federal court if the case could have been brought originally in federal court. This case could have been brought originally in federal court because the engineer was a citizen of State C and the corporation was a citizen of States A and B, and the amount in controversy exceeded $75,000. The engineer may add together all of the engineer’s claims, even two unrelated claims, to get over the amount in controversy threshold. However, there is an exception to the right of removal that applies here. A corporation who is a citizen of the state where the case is brought originally may not remove a case, even if the case could have been brought originally in federal court, if the case if founded solely on diversity (as it was here) and the corporation is a citizen of the state where the case was brought. Here, the corporation was a citizen of State A because it has its principal place of business there, under 28 U.S.C. Section 1332(c)(1). Therefore, under 28 U.S.C. Section 1441(b), the corporation may not remove the case to federal court.

118
Q

Class certification and immediate appeal

A

Federal Rule of Civil Procedure 23(f) allows an appeal from an order granting or denying class-action certification if a petition for permission to appeal is filed within 14 days after the order is entered. A good discussion of this topic by the U.S. Supreme Court appears in Microsoft Corp. v. Baker [582 U.S. __ (2017)].

119
Q

Introducing withdrawn plea

A

Pursuant to Federal Rule of Evidence 410, a plea that is later withdrawn may not be admitted into evidence against the defendant who made the plea (or participated in the plea discussions) in any civil or criminal proceeding. Because the man withdrew his guilty plea, that plea may not be introduced against him in a subsequent civil proceeding.

120
Q

Jurors testifying

A

Generally, jurors may not testify about matters occurring during deliberations, but there are exceptions to this general rule where extraneous prejudicial information was brought to the jury’s attention, an outside influence was improperly brought into jury deliberations, or there was a mistake in entering the verdict on the verdict form. Because the juror improperly brought extraneous information (the newspaper article) into the jury deliberations, a juror would properly be able to testify regarding the discussion of the newspaper article to determine the validity of the verdict.

121
Q

Right to counsel in misdemeanors

A

The right to counsel in a misdemeanor trial applies when a sentence of incarceration is actually imposed. Absent waiver, a defendant may not be imprisoned for any offense, whether classified as petty, misdemeanor, or felony, unless he was represented by counsel. Note that a fine may be constitutionally imposed on the defendant even if denied the right to be represented by counsel. The rule only applies to misdemeanor cases where a sentence of incarceration is actually imposed. It does not have any impact on any fine imposed by the court.

122
Q

embezzlement

A

Embezzlement is the fraudulent conversion of the property of another by one already in lawful possession.

123
Q

using a drug sniffing dog outside a house

A

In Florida v. Jardines [133 S. Ct. 1409 (2013)], the Supreme Court held that using a drug-sniffing dog to investigate around and near the front door of a home constituted a search for Fourth Amendment purposes. Because using the drug-sniffing dog in this context is a search, the officers must have probable cause and a warrant. Here, the narcotics officer did not have probable cause or a warrant to bring the dog to the front of the house. Thus, the chemist’s Fourth Amendment rights were violated.

124
Q

public employment and political membership

A

An individual cannot be denied public employment based upon membership in a political organization [Keyishian v. Board of Regents, 385 U.S. 589 (1967)]. Because the university did not renew her contract based solely upon her membership in the radical political party, it violated the lecturer’s First Amendment rights.

125
Q

obscenity test

A

Obscenity is not constitutionally protected speech and can be regulated. However, under Miller v. California [413 U.S. 15 (1975)], material must meet the following three parts of this test to be considered obscene: (1) the material, taken as a whole, must appeal to the prurient interest, applying contemporary community standards; (2) the material must depict in a patently offensive way the sexual conduct specifically defined by a state statute; and (3) the material must lack serious literary, artistic, political, or scientific value (using a national reasonable person standard, pursuant to Pope v. Illinois [481 U.S. 497 (1987)]). Here, as the magazine was modeled after a critically well-received art show, the customer’s best argument is that, based on a national reasonable person standard, the magazine has serious artistic value and is therefore not obscene.

126
Q

Private nuisance

A

A private nuisance is an activity or a thing that substantially and unreasonably interferes with a plaintiff’s use and enjoyment of his property. Unreasonableness is assessed by weighing the gravity of the harm against the social utility of the activity. There are no facts to conclude that there has been either a substantial interference or an unreasonable interference. The group home is in full compliance with zoning requirements and the resident’s singing does not substantially interfere with the neighbor’s use and enjoyment of his property because the facts indicate his singing occurs only occasionally and only for several minutes. The facts do not support a conclusion that the neighbor has been harmed in any way.

127
Q

intentional torts and respondaet superior

A

Under the respondeat superior doctrine, an employer is liable even for the intentionally tortious acts of his employees, so long as they are committed within the scope of employment. Here, although the employee’s actions were intentionally tortious, they were closely connected and related to the shop business and were therefore within the scope of her employment. As such, the employer will be vicariously liable for the employee’s act.

128
Q

defamation and police

A

The defendant will want to eliminate his liability for defamation, or at least want to point to facts that will subject the plaintiff to proving a higher standard (i.e., the standard that applies to defamation of a public official). These are facts the defendant would want to point out to support his claim that a police patrolman is a public official to whom the Times v. Sullivan [376 U.S. 254 (1964)] standard applies.

129
Q

reputable manufacturer or distributor

A

Regarding a products liability case based on a negligence theory, in general, a failure to inspect packaged goods for defects is not a breach of duty if they come from a reputable manufacturer or distributor. If the goods are manufactured or otherwise supplied by a previously unknown or questionable source, defendant’s unreasonable failure to inspect is a breach of duty. It would, however, probably be a breach of duty not to inspect a particular product as to which defendant had received or become aware of customer complaints. In this case, the retailer purchased the blenders from a manufacturer with a reputation for producing high-quality goods. Therefore, the retailer’s failure to inspect the packaged blenders is not a breach of duty, and this answer choice is correct. Note that the retailer could be liable on a strict liability theory because it is in the distribution chain, even if it is without fault.

130
Q

Obviously dangerous for warnings

A

It is true that a product may be considered dangerously defective when it is accompanied by an inadequate warning. However, certain products (such as knives) are regarded as so obviously dangerous that a warning is considered unnecessary. Here, the products at issue are Roman Candle fireworks, which are obviously dangerous. The court will likely rule that no warning is necessary, and therefore, the plaintiff will not prevail in a products liability action based on failure to provide adequate warning.

131
Q

employer’s privilege for defamation

A

An employer has a conditional privilege against defamation when he or she comments on an employee’s performance at the request of a prospective employer. This privilege exists when a defendant communicates on a matter of interest to the recipient. This answer choice correctly states the law–the partner is not liable if his statement was based on a reasonable belief. The privilege, because it is conditional, can be defeated by a showing that the employer’s opinion was based on actual malice, that he exceeded the scope of the privilege, or that he does not believe the communication to be true. If any of these applies, the defendant is liable for defamation. On the other hand, if his opinion was based on reasonable (albeit mistaken) grounds, his opinion is not actionable defamation. Because it does not appear that the defendant’s opinion was based on malice, that he exceeded the scope of the privilege, or that he did not believe the statement to be true, he is not liable.

132
Q

liability on custodians for nonfeasance in wrongful death suicide actions

A

Remember that for exam purposes, joint and several liability, with pure comparative negligence, is the relevant rule unless otherwise indicated. A plaintiff’s recovery will be reduced by the degree of his fault. The facts clearly state that the police were negligent in failing to prevent the suicide. Courts have imposed liability on custodians for nonfeasance in wrongful death suicide actions (usually involving hospitals and police departments). Note that custodians are liable for nonfeasance with respect to suicide if they had reason to know of a particular individual’s suicidal tendencies and failed to take reasonable protective steps.

133
Q

Civil malicious prosecution

A

To prevail on malicious prosecution, the case against the plaintiff must have terminated in a manner indicating the plaintiff’s innocence. Malicious prosecution is the institution of criminal proceedings by a defendant, done for an improper purpose, and without probable cause, which terminate favorably to the plaintiff, and which cause the plaintiff damages. The criminal prosecution must have terminated in a fashion indicating that the plaintiff was innocent of the charges. Termination on the merits (i.e., an acquittal after trial or court dismissal for lack of sufficient evidence) is sufficient in this regard; termination based on procedural or technical defects, prosecutorial discretion, or similar grounds is not. A mistrial resulting from juror misconduct is a technical defect that does not indicate the gang member’s innocence. Therefore, the gang member should not prevail in his action.

134
Q

tort of intrusion into seclusion again

A

The tort of intrusion into seclusion requires the intentional intrusion upon the solitude of another and that the intrusion be highly offensive to a reasonable person. In this case, both elements are present. The photographer clearly intruded upon the mayor’s solitude, and his intrusion would be highly offensive to a reasonable person. The situation was also one in which the mayor had a reasonable expectation of privacy. As such, the mayor is likely to prevail.

135
Q

sof main purpose exception

A

The general rule is that a promise to answer for the debt of a third party is subject to the Statute of Frauds. However, under the main purpose exception, if the main purpose of the guarantor’s promise is to protect or promote his own economic interests, rather than the interests of the debtor, then the agreement is not within the Statute of Frauds and no signed writing is required. Under this exception, if the shareholder (i.e., the promisor) sought to act as surety for the firm’s loan for his own economic advantage (as 25% shareholder), then his oral promise to guarantee the loan will be enforceable.

136
Q

strict products liability

A

Strict products liability requires: (1) a proper plaintiff who is injured using the defective product, or another foreseeable plaintiff such as a bystander or rescuer; (2) a proper defendant in the chain of distribution; (3) a manufacturing defect, a design defect, or a failure to warn; (4) causation; and (5) damages.

137
Q

conspiracy v solicitation

A

In this question, students are being tested on the distinction between solicitation and conspiracy. For the crime of solicitation to be completed, it is only necessary that the defendant entice, advise, incite, order, or otherwise encourage another person to commit a crime with the intent that the crime be committed. On the other hand, conspiracy requires an agreement between two or more individuals to commit a criminal offense. Under the given facts, no true agreement to commit a battery actually occurred. Rather, the girlfriend solicited the boyfriend to recklessly engage in conduct, knowing that it would create a high risk of causing injury to the patrons in the movie theater. Thus, the girlfriend and boyfriend would be guilty of battery, but not of conspiracy.

138
Q

difference between settlement negotiations and an offer to pay expenses resulting from an injury

A

Step 1: Under Fed. R. Evid. 409, evidence of furnishing or offering to pay medical, hospital, or similar expenses resulting from an injury is not admissible to prove liability for the injury. Statements made by an opposing party, under Fed. R. Evid. 801, are not hearsay and are admissible against the speaker.

Step 2: In this fact pattern, the boyfriend made both an offer to pay medical expenses and an admission of fault. The offer to pay medical expenses is inadmissible under Rule 409, while the admission of the party-opponent will come in. Rule 409 does not help the boyfriend here, because that the admission of possible fault had nothing to do with medical expenses. Select this answer.

139
Q

Deed acceptance

A

Acceptance is presumed if the gift is beneficial.

140
Q

rules that discriminate against religious behavior

A

Answer (C) is correct because the amendment is neutral and does not discriminate against religion. The Supreme Court has held that a law may violate the right to free exercise of religion if it discriminates against religious conduct [Church of the Lukumi Babalu Aye, Inc. v. City of Hialeah, 508 U.S. 520 (1993) (striking down a law that prohibited sacrificing animals for religious purposes)]. In Lukumi Babalu, the law discriminated against religious conduct because it allowed the killing of animals for non-religious purposes, but prohibited killing animals for religious reasons. On the other hand, the Supreme Court has held that free exercise rights cannot be violated by laws that restrict or burden religious conduct but that are neutral and generally applicable [Employment Division v. Smith, 494 U.S. 872 (1990) (upholding a law that prohibited the use of certain drugs, even though those laws imposed a burden on those seeking to use the drugs for religious purposes)]. In Smith, the law did not discriminate against religion because it prohibited all use of the drugs, not just use of the drugs for religious purposes. In this question, the city amended its employee benefit plan in a neutral and generally applicable way. The amendment precluded benefits from being paid if the employee refused to accept medical care for any reason other than excessive risk to life or health. The exclusion applied if the person’s reason for refusing medical care was motivated by religion, but it also applied if the person’s reason for refusing medical care had nothing to do with religion. The amendment therefore did not discriminate against religion, and it cannot violate free exercise rights.

141
Q

chances of survival rule

A

Jurisdictions that allow recovery for the loss of the chance of survival have created an exception to the traditional common law rules for establishing cause in fact. Under the traditional rules, the wife would be required to prove that reasonable action on the part of the hospital (presumably a correct diagnosis) would, more likely than not, have led to the man’s survival. Here, the wife cannot establish that the chances of the man’s survival would have been greater than 50% even if he had been given appropriate medical care. A jurisdiction that allows recovery for loss of the chance of survival, however, would allow the wife to recover for the reduction in her husband’s chance of surviving that was caused by the failure to properly diagnose.

142
Q

about interrogatories

A

A party may not seek discovery from any source before the parties have conferred for a pretrial conference and planned for discovery [Fed. R. Civ. P. 26(d)(1)]. Because there has clearly been no pretrial conference, the plaintiff’s attorney could not serve the defendant with the interrogatories.
interrogatories may cover more than simple facts, including questions that ask for contentions or opinions.

143
Q

police can lie to enter

A

The consent here was voluntary: nothing the officer did coerced the defendant to consent to the entry. Furthermore, the Supreme Court has held that consent is not invalidated if police use undercover tactics to gain entry into a home by pretending to be customer seeking to engage in a criminal transaction. This means the officer’s presence in the home was lawful.

144
Q

owner’s title insurance policy coverage

A

An owner’s policy of title insurance only protects the owner of the policy and does not run with the land to subsequent owners.

145
Q

late service

A

Because the buyer’s attorney did not effect timely service, the only way to survive the seller’s motion to dismiss is for the buyer to demonstrate good cause for failing to do so. Fed. R. Civ. P. 4(m). Forgetting to effect service is not good cause, which typically includes something beyond the party’s control.

146
Q

substitute work and duty to mitigate

A

This question involves the principle of mitigation of damages. Upon a breach of contract, the aggrieved party may not recover for any losses it could have reasonably avoided. In the employment context, an aggrieved employee need only make reasonable efforts to secure a position that is reasonably equivalent to the job lost and, accordingly, she need not accept substitute work when it is in a different field, or offers significantly lower pay or less desirable terms and conditions of employment than those of the lost job. In this case, the brick mason was not required to take the job of either night guard or day laborer, which paid substantially less than his brick mason job. Therefore, he may recover the full $45,000 contract price in damages. (A) is therefore incorrect because the $25,000 in other, lesser employment will not reduce the brick mason’s recovery. (C) is wrong because the builder’s lack of bad faith will not prevent the brick mason from recovering for the damages causes by the builder’s breach of contract. Finally, (D) is wrong because, as discussed above, the brick mason was not required to take the lesser employment offered in order to mitigate his damages.

147
Q

right to a jury trial

A

If a right to a jury trial exists in an action, any party can exercise the right by filing with the court and serving on the other parties a written demand for a jury trial [Fed. R. Civ. P. 38]. The demand for a jury trial must be made within 14 days after the service of the last pleading directed to the triable issue [Id.]. Here, because the architect has not yet filed an answer to the first or second complaint, the last pleading has not been served on any triable issues. Consequently the 14-day clock has not started running yet.

148
Q

Misuse for product liability

A

Misuse is a defense to a strict products liability action. Misuse requires a use in a manner that is neither intended nor foreseeable. Here, using a stove to heat a lava lamp to make the fluid inside move faster is a misuse, and it is not a foreseeable one. Consequently, the collector will not prevail because the manufacturer has a valid defense.

149
Q

Assumption of risk in product liability

A

The defenses to strict products liability are misuse, assumption of risk (to a limited extent), and either contributory negligence or comparative negligence, whichever applies in the relevant jurisdiction. Assumption of the risk is only a defense in strict products liability where the plaintiff: (1) knew of the defect; (2) understood the risks posed by the defect; and (3) voluntarily elected to expose herself to those risks. Here, there is no indication that the collector was aware of the risk that the lamp could explode if too much heat was applied. Therefore, she could not be said to have assumed the risk.

150
Q

manufacturer liability for components

A

A person who assembles component parts into a finished product is strictly liable for defects in the components used. The manufacturer of the component part is also strictly liable for defects in the component. In this case, the scooter producer is in the chain of distribution, so it will be liable, even though the scooter producer was not at fault for the defective wheel.

151
Q

known risks

A

However, a seller is not required to warn with respect to products which are only dangerous when consumed in excessive quantity or over a long period of time, when the danger is generally known and recognized.

152
Q

baby witness competence

A

Under FRE 601, every person is competent to be a witness. FRE 602 limits witnesses to testifying as to matters within their personal knowledge. A witness with personal knowledge of the accident will be competent to testify, even if that witness is a child. At the common law, witnesses would be competent only if they both possessed personal knowledge of the subject matter of their testimony and possessed the capacity to perceive and tell the truth. However, these latter requirements have been abandoned by the FRE. Under the FRE, questions of mental competence (based on age, illness, drug use, etc.) go to the weight of the evidence, and not admissibility. Thus, the child is a competent witness in this action, and this answer choice is correct.

153
Q

judicial notice of expert testimony

A

Under Federal Rule of Evidence 201, this type of evidence is admissible. Where the tester is qualified and there is adequate foundation, a court may take judicial notice. This type of testing is sufficiently reliable, given the state of modern technology, to be the subject of judicial notice. Therefore, this is the best answer.

154
Q

superceding when worker jumps

A

the worker’s decision to jump out of the elevator, even though he was not in danger, was unforeseeable, and thus was a superseding cause of the injury that terminated the manufacturer’s potential liability. This question is similar to the scenario in Egan v. A.J. Constr. Corp. [724 N.E.2d 366 (N.Y. 1999)]. The elevator manufacturer in that case successfully argued that the worker’s decision to jump out of the elevator was so unexpected and unforeseeable that it broke the chain of proximate causation between the elevator’s defect and the injuries. As a result, the elevator’s defect was not a legal cause of the injuries. Even if the manufacturer was not able to avoid liability completely based on this argument, this answer would also be correct because comparative fault would enable the manufacturer to at least reduce the liability to some extent. The old, traditional rule was that contributory negligence could not be used as a defense to a strict tort liability claim against the manufacturer of a defective product. However, most states have now replaced contributory negligence with some form of comparative fault, and they have held that comparative fault is a defense that can be used against a strict products liability claim. On the multistate portion of the bar exam, you must assume that pure comparative fault applies unless the question indicates otherwise. The elevator manufacturer would therefore be able to argue that because the worker got impatient and jumped out of the elevator, his injuries were at least partially his own fault, and the liability should be reduced by the share of the fault that the jury assigns to the worker. So even if the worker was able to recover something, it would not be a recovery for 100% of the damages for his injury.

155
Q

commercial mortgage prepayment restrictions

A

In a mortgage secured by commercial property, both prepayment prohibitions and prepayment fees are valid and permissible.

156
Q

can car occupants be ordered out of the car

A

The stop of the car was constitutional, because it was objectively justifiable (regardless of the officer’s subjective motivation), and both the driver and any passengers may be ordered to step out of a car during a lawful traffic stop.

157
Q

compelling unwilling witness

A

A witness cannot be compelled to provide potentially incriminating testimony unless the witness is granted use and derivative-use immunity.
but the witness need not be granted transactional immunity.

158
Q

corporation and fifth amendment privilege

A

The Fifth Amendment privilege against compelled self-incrimination applies to both the federal and state governments and prohibits compelling evidence of at testimonial or communicative nature where the individual compelled has a real and substantial fear responding to the government question will result in self-incrimination in the United States. Production of documents in response to a grand jury subpoena can implicate the privilege because the documents may contain evidence of a testimonial or communicative nature. However, the privilege belongs to individuals and not corporations. According to United States v. White [401 U.S. 745 (1971)], the privilege is “limited to its historic function of protecting only the natural individual from compulsory incrimination through his own testimony or personal records.” Even if the subpoena is directed to an individual serving as a corporate officer, that individual is considered to hold the documents in a representative and not individual capacity.

159
Q

objecting to the jury instructions

A

A party who objects to a jury instruction must do so before the challenged instruction is given to the jury [Fed. R. Civ. P. 51(c)(2)(A)]. Because the defendant did not do so, the issue has not been preserved for appeal.

160
Q

when must jmol be submitted

A

A motion for a judgment as a matter of law (JMOL) is granted if the court finds that a reasonable jury would not have a legally sufficient evidentiary basis to find for the nonmoving party on the issue at hand [Fed. R. Civ. P. 50(a)]. A JMOL must specify the judgment sought and the law and facts that entitle the moving party to the judgment. Motions for JMOL are typically made at the close of the nonmoving party’s case, but they may be made at any time after all of the nonmoving party’s evidence is submitted. However, the motion must be made before the case is submitted to the jury. If a party has timely moved for JMOL, that party may serve a renewed JMOL to set aside a later verdict [Fed. R. Civ. P. 50(b)]. However, if a party does not timely move for a JMOL, that party may not move for a renewed JMOL. Here, because the wholesaler did not raise the issue in a JMOL before the case went to the jury, the wholesaler cannot move for a renewed JMOL.

161
Q

civil judgment based on a criminal conviction

A

The stockbroker’s motion is made under Federal Rule of Civil Procedure 60(b)(5), which allows the court to grant relief from the final judgment in the civil action because the criminal judgment on which the civil judgment was based has been reversed and vacated.

162
Q

witness with no memory of prior identification

A

The U.S. Supreme Court held in United States v. Owens that a declarant-witness is subject to cross-examination within the meaning of the hearsay exception for prior identifications even if the witness lacks memory of the prior identification. As to personal knowledge, the evidence of defensive wounds is more than sufficient to persuade a reasonable juror that the victim saw his attacker.

163
Q

residuary legatee

A

If a testator leaves a specific piece of property in a will, and that piece of property is no longer in the estate, the bequest fails through the doctrine of ademption. Here, the woman’s bequest fails because the woman specifically left the farm to her nephew, but the farm was no longer in her estate. Consequently, the nephew receives nothing, and the niece receives the home because she is the residuary legatee and receives whatever is left in the estate.

164
Q

continued duty to disclose and misrepresentation

A

Misrepresentation is a defense to contract. Although the misrepresentation can be affirmative, fraudulent nondisclosure of a material fact is also available as a defense where the nondisclosing party had a duty to disclose the concealed fact and failed to do so. Although there is generally no duty of disclosure to trading partners, if a party is aware of material facts that are unlikely to be discovered by the other party in the exercise of ordinary care and diligence, then there will be a duty to disclose that information if a party has made an assertion that was true at the time but has been rendered untrue by intervening events. Thus, because the woman had provided information about her financial affairs, which had since drastically changed, she committed fraudulent nondisclosure by failing to update the bank on the changed state of affairs. (A) is wrong because affirmative misrepresentations are not always required for availability of the defense where fraudulent nondisclosure is applicable. (B) is wrong because it ignores the woman’s duty to update the bank on the changed state of affairs. (D) is wrong because a longstanding relationship of trust and confidence is not necessary to impose the duty of disclosure where a party has made an assertion that was true at the time but has since been rendered untrue by intervening events (although a relationship of trust and confidence can be an alternative basis for imposing the duty of disclosure).

165
Q

reviewing unconstitutionally excessive punitive jury award

A

When a trial court reduces a punitive damages award on the ground that it is unconstitutionally excessive, the appellate court reviews that determination de novo. See Cooper Industries, Inc. v. Leatherman Tool Group, Inc., 532 U.S. 424 (2001).

166
Q

Purchase money mortgage on real estate precedence

A

The bank’s mortgage is a purchase-money mortgage, meaning that the funds the bank advanced were used to purchase the land. A purchase-money mortgage executed at the same time as the purchase of the real property encumbered takes precedence over any other claim or lien, including a previously filed judgment lien. Therefore, the bank’s purchase-money mortgage takes precedence over the credit card company’s judgment lien.

167
Q

trial plan issue

A

Under Federal Rule of Civil Procedure 16(e), once a trial court issues an order regarding the trial plan after a final pretrial conference, that order may be modified only to prevent manifest injustice. Thus, if the manufacturer’s attorney wants to modify the final pretrial order, the attorney must move for a modification and demonstrate that manifest injustice would result if the order is not modified.
An immediate appeal from the final pretrial order is not possible because there is no final judgment in the action. Further, if the manufacturer’s attorney does not seek to modify the final pretrial order at the trial-court level, the issue will not be preserved for appeal. Thus, the attorney should move to modify the order by demonstrating that manifest injustice would result if it is not modified. Fed. R. Civ. P. 16(e). If the court rejects that motion, then the attorney will be able to challenge the court’s ruling on appeal, where it will be reviewed for an abuse of discretion. See United Phosphorus, Ltd. v. Midland Fumigant, Inc., 205 F.3d 1219, 1236 (10th Cir. 2000).

168
Q

chemical strict liability

A

Answer (D) is correct because the chemical company would be subject to strict liability for harm resulting from the ultrahazardous, abnormally dangerous activity of storing the toxic chemicals. Strict liability applies when a person engages in an activity that is inherently and highly dangerous no matter how carefully the activity is done. For example, if someone uses explosives to blast through rock for a tunnel or building foundation, the person is strictly liable for damage done by the blasting, even if the person is extremely careful about how the blasting is done and takes every reasonable precaution to prevent harm from occurring. Likewise, strict liability would apply where the chemical company is storing a toxic chemical that creates a serious risk of harm even when reasonable care is used.
Answer (C) is incorrect because the chemical company is strictly liable for harm resulting from its storage of the toxic chemicals, but would not necessarily be liable for any and all harm caused by the chemicals. If the toxic chemicals were a dangerously defective product that the chemical company produced and sold, then the chemical company could be strictly liable under products liability law for any harm caused by the chemicals. However, this question is not about a products liability scenario. The chemical company is not in the business of selling the toxic chemicals, and instead, the toxic chemicals are merely an unwanted by-product that the company will send to a disposal facility. The chemical company is subject to liability because it is engaged in the ultrahazardous, abnormally dangerous activity of storing a toxic chemical at its plant. The company is therefore strictly liable for harm resulting from that activity. The company would not be strictly liable beyond that, so it is an overstatement to say the company would be liable for any harm caused by the chemicals. For example, if the chemical company transferred control of the toxic chemicals to a business that specialized in disposing of such chemicals, the chemical company would no longer be engaged in an ultrahazardous activity once it was no longer storing or controlling the chemicals, and the chemical company would not be subject to strict liability for anything that happened to the chemicals at a later time and place. If you picked this answer choice, you were on the right track and correctly recognized that strict liability would apply here, but this answer overstates the extent of the chemical company’s potential liability a little bit and therefore is not quite the best answer choice. This is a good example of how questions may have more than one answer choice that are pretty good, and your challenge is to pick the very best one.

169
Q

robbery and or larceny

A

Robbery or larceny but not together. It is true that all the elements of larceny and robbery (which may be thought of as aggravated larceny) were present. The woman’s threat of immediate harm to the clerk was sufficient to constitute the intimidation required for robbery. However, because larceny is a lesser included offense of robbery, she cannot be convicted of both offenses for a single incident.

170
Q

easement by necessity again

A

An easement implied by necessity requires: (1) severance of title of land held in common ownership; and (2) strict necessity for the easement at the time of severance. Here, the man owned both tracts of land and conveyed the eastern half to the woman. By doing so, he landlocked his property with no access to the public road. This created an easement by necessity from the man’s property over the woman’s property. He then left the property to his cousin. While the cousin could obtain other ways off the property, he is within his rights to sue for access over the original easement.

171
Q

withdrawal under common law

A

A bilateral conspiracy requires proof of three material elements: the actus reus is the actual agreement between two or more parties (which can be established by direct or circumstantial evidence); the mens rea is the intent to agree and the intent to commit a target offense (here the offense of robbery); and the attendant circumstance is an overt act committed by one of the co-conspirators in furtherance of the agreement, even if it is trivial and quite distant from the actual target offense. This question indicates all these elements were met when the defendant entered into the criminal agreement and then brought the car to the agreed upon location. Note that the question does not ask whether the defendant can be convicted of the target offense of robbery, but only of the conspiracy to commit robbery. A conspiracy is a “stand alone” crime and does not merge with the attempted or completed target offense (unlike attempt). Once the elements of conspiracy are established beyond a reasonable doubt, the defendant is guilty of the conspiracy. At common law, a defendant could not renounce his guilt for the conspiracy by withdrawing after the overt act indicated the conspiracy was formed (in other words, the defendant could not “put the toothpaste back in the tube”). A defendant could withdraw by notifying all co-conspirators he would no longer participate, but this has no impact on whether he is guilty of the conspiracy; it only severs the defendant from conspiracy liability for subsequent crimes committed in furtherance of the conspiracy. In this question the defendant may have tried to withdraw by leaving the note but his efforts were irrelevant to the question of guilty for the conspiracy because: (1) even if he successfully withdrew it does not function to erase his guilt for the conspiracy he had already entered into; and (2) because his co-conspirator would not have seen the note until after the robbery it would not even qualify as an effective withdrawal (which means he could also be convicted of the robbery).

172
Q

opportunity to be heard when criminal conviction is basis for license action

A

Answer (C) is correct because there is no need for a hearing if the relevant facts are already indisputably established. The Constitution guarantees that the government will not deprive people of life, liberty, or property without due process of law. Due process generally means that a person receives notice and an opportunity to be heard. This enables the person to challenge the validity of the government’s action. However, if there are no disputed issues, then having a hearing serves no purpose and is not required. For example, if there are no genuinely disputed factual issues in a civil lawsuit, a judge might resolve the case by granting summary judgment, because there is no need for conducting a trial. Likewise, if there are no disputed issues relating to the revocation of a professional license, there may be no need for a hearing about it. In this question, the doctor does not deny the criminal conviction or the factual basis for the conviction. If those facts are settled and unchallenged, and they leave no possible outcome other than revocation of the doctor’s license, then due process does not require a hearing.

173
Q

adult standard of care for teenager

A

Answer (D) is correct because the girl will be held to an adult standard of care for operating a high-speed motorboat. Children generally are required to act with the care of a reasonable child of the same age, intelligence, maturity, and experience. For example, this means that a 13-year-old would generally be required to act with the care of a reasonable 13-year-old. However, the adult standard of care applies when a child is engaged in activities that are inherently somewhat dangerous and typically are done only by adults. Courts often apply this rule to children operating motorized vehicles, such as automobiles, airplanes, or boats. A 13-year-old operating a high-speed motorboat would therefore be held to the adult standard of care and would be expected to act with the care of a reasonable adult, not just the care of a reasonable 13-year-old.

174
Q

duty to warn about open and obvious dangers

A

Answer (D) is correct because a property owner generally does not have a duty to warn about a danger that is open and obvious. The question says that the jurisdiction applies the traditional rules of landowner liability, under which the landowner’s duty varies depending on the status of the person who was injured. If the property was held open to the public or if the person was there for business dealings with the landowner, the person would be an invitee, and the landowner would owe a duty to act with reasonable care in keeping the property reasonably safe for the invitee. If the person was a mere licensee, who had permission to be there but was not an invitee, the landowner’s duty would be less extensive. The landowner would merely need to warn and protect the licensee against dangers known to the landowner. If the person was a trespasser on the property, the landowner generally would owe no duty. However, even where the landowner owed some duty, because the person was an invitee or licensee, the landowner generally would not have a duty to warn someone about a danger that was obvious or already known to the person. There is no point in warning about an obvious or known danger, because it does no good to tell someone about something that they can plainly see or that they already know about. There may be exceptional circumstances where a landowner would have a duty to warn, even though the danger was obvious, such as if the landowner has reason to foresee that someone is likely to be hurt despite the obviousness of the danger. But absent some special reason why a warning will be useful, there is no duty to warn about what is already obvious and known to the person. In this question, the grandfather is a licensee, because he has permission to be in the house, but he is not there for a business purpose. The question specifically notes that the grandfather regularly visited the house and was aware that toys were routinely left scattered on the floor. The presence of the toys on the floor was open and obvious, not a latent or hidden danger. Absent some special reason to think his father might be harmed despite the fact that the danger was obvious and the grandfather was well aware of it, the son had no duty to give a warning.

175
Q

limitation on contract damage

A

A limitation on recovery of damages for breach of contract is foreseeability. That is, the aggrieved party may not recover damages caused by the breach unless, at the time of contracting, the breaching party knew or had reason to know that the particular consequential damages sought would result from breach. The supplier had no reason to know about the $1,000-per-day early bonus, and therefore will not be liable for those damages. (D) is therefore wrong because there was no factual reason for the supplier to know (also, time is generally not of the essence unless the contract is for the sale of goods, or the contract specifically so provides). (C) is wrong because it assumes all damages caused by the breach are recoverable, but this ignores the foreseeability limitation. (A) is wrong because the fact that the supplier did not expressly agree to be responsible is not the determinative factor; rather, the fact that the damages were not foreseeable is. Had the damages been foreseeable, they would have been recoverable even in the absence of an express agreement to be responsible for them.

176
Q

Rescue doctrine for negligence

A

Under the “rescue doctrine,” a tortfeasor who creates a dangerous situation owes an independent duty of care to the rescuer, which arises even when the defendant endangers no one’s safety but his own. The original wrongdoer remains liable for the harm he causes, regardless of whether the rescuer succeeds and whether the rescuer injures himself, the person rescued, or a stranger. Here, it was reckless for the kayaker to enter the water that was still swollen due to the spring thaw, and it was foreseeable that someone would try to rescue the kayaker once he lost control of his craft. The kayaker is liable to the fisher for her injuries.

177
Q

Police report admissibility

A

A distinction must be made between the actual police report and the contents contained within the police report. The police report is admissible as a public record, but anything within the report related to factual findings (such as the defendant having stolen the car) must be excluded.

178
Q

Testimonial statement by unavailable witness

A

In Crawford v. Washington [541 U.S. 36 (2004)], the U.S. Supreme Court ruled that the Confrontation Clause applies to hearsay statements that are testimonial in nature where the hearsay declarant does not testify at trial and the accused did not have any prior opportunity to cross-examine the declarant. Unless the defendant was given a prior opportunity to cross-examine the woman, the statements to the officer are inadmissible.

179
Q

Treatise as substantive evidence

A

Rule 803(18) of the Federal Rules of Evidence provides that statements contained in treatises may be admitted into evidence during direct or cross-examination of an expert witness if: (1) the treatise is established as a reliable authority; and (2) the treatise is called to the attention of the expert witness during cross-examination or is relied upon by the expert in direct testimony. As an exception to the hearsay rule, statements in treatises admitted pursuant to Rule 803(18) may be used as substantive evidence, by reading them to the jury.

180
Q

babysitter’s privilege

A

A nonparent who has been legally given or has voluntarily assumed the control, training, or education of a child is privileged to apply such reasonable force or to impose such reasonable confinement as she reasonably believes to be necessary. A parent may, however, restrict the privilege of one to whom she has entrusted the child. Here, the babysitter was hired to babysit the boy, and there is no indication that the mother restricted the babysitter’s ability to confine the boy as to a time out. As long as the babysitter reasonably believed that the boy’s confinement was necessary, and so long as she used reasonable force to impose such confinement, the babysitter’s actions will be privileged. Given that the facts indicate the babysitter believed that the boy needed to be confined, she will most likely be successful in asserting that her actions were privileged.

181
Q

Strict liability for trespassing livestock

A

A defendant will be held strictly liable for any trespass to the land or chattels of a plaintiff by wild animals of a kind likely to escape, trespass, and do damage, or by livestock possessed by the defendant. In this case, the herder’s sheep are livestock, and they trespassed on the farmer’s property, causing damage. Therefore, the herder will be strictly liable, and this answer choice is correct.

182
Q

abnormal sensitivity and strict liability

A

Strict liability may be imposed on one who carries on an abnormally dangerous or ultrahazardous activity. The entity will be liable for harm resulting from the activity even if he exercised the utmost care to prevent the harm. The plaintiff must prove, however, that the defendant’s abnormally dangerous activity was both an actual and proximate cause of the plaintiff’s harm. This means the defendant will be strictly liable only if the harm results from the risk that makes the defendant’s activity abnormally dangerous. Strict liability will not be imposed if the harm occurred only because of the abnormal sensitivities of the plaintiff’s activities. Thus, this answer choice is correct.

183
Q

strict liability defenses

A

All three of these (misuse, alteration, and assumption of the risk) may be available to a defendant to defend against a strict products liability claim. If a plaintiff uses a product in a manner that is neither intended nor foreseeable, he has misused the product and it cannot be defective. Further, a manufacturer or seller may have their liability reduced (or in a contributory negligence jurisdiction, relieved entirely) for a product where the product is altered (changes in design, formula, function, etc.) after leaving its hands. The alteration must be substantial in order to relieve the manufacturer or seller of liability. Additionally, if a plaintiff knows of the defect, comprehends the risks posed by the defect, and voluntarily elects to expose himself to those risks, the plaintiff will be deemed to have assumed the risk.

184
Q

no warning required for obviously dangerous products

A

A product may be considered dangerously defective when it is accompanied by an inadequate warning. It may fail to sufficiently describe the danger, fails to mention all dangers, or is inconsistent with the instructions for use of the product. Where the plaintiff establishes that the manufacturer of a product knew or reasonably should have known of a danger presented by the product and failed to take the precautions a reasonable person would have taken to warn adequately of that danger, the absence of such warning is sufficient to impose strict liability. In this case, the axe did not have a warning that it was dangerous. However, the danger of certain products is so obvious that a warning is unnecessary. Here, the axe did not require a warning, because it is obviously dangerous. As such, the company will prevail.

185
Q

defect discoverability in reasonable inspection

A

The potential danger involved in the use of snowmobiles makes it necessary for the manufacturer to inspect its components and the final product. For defective products, a plaintiff suing in negligence must show the existence of the defect and that the defect would have been discoverable upon reasonable inspection. If a reasonable inspection would have found the defect, and the supplier did not reasonably inspect, this would lead to a finding for the plaintiff under a negligence theory of product liability.

186
Q

Physician-patient privilege

A

The physician-patient privilege applies when the patient consults the doctor for purposes of diagnosis or treatment. Here, friend A went to the doctor to seek medical advice, and friend A discussed his narcolepsy and the auto accident only in response to the doctor’s questions. Given that the conversation between the doctor and friend A was for the purpose of diagnosis and treatment, the physician-patient privilege applies. According to the physician-patient privilege, a patient has a privilege to refuse to disclose and to prevent any other person from disclosing confidential communications made for the purpose of diagnosis or treatment. Friend A, the patient, has the privilege to prevent the doctor from disclosing the communications at trial. While it may appear that the presence of friend B should destroy the confidentiality of the communication, it does not. The mere presence of a third party does not automatically destroy the physician-patient privilege when the person is present: (1) to aid the patient; (2) as a necessary and customary participant in the consultation or treatment; (3) to assist the physician or enable the patient to obtain treatment; or (4) where the information is readily discernable to everyone present. Here, friend B was present because he believed he could provide additional necessary information which would aid the patient in obtaining treatment. As such, his presence was to aid the patient, and his being there will not destroy the physician-patient privilege.

187
Q

Federal common law for evidence in federal questions

A

Under Federal Rule of Evidence 501, for cases in federal court when the issues involve federal law, the application of privilege is determined based upon common law determined by United States courts. Here, the flight attendant is claiming discrimination in violation of federal law, therefore, federal common law will apply. Therefore, this answer is correct.

188
Q

Adequate and independent state grounds note

A

:Where a state court’s decision is based upon its interpretation of a similar federal law, adequate and independent state grounds will not apply. The Supreme Court may review in this situation. This is the case at hand, because the state law is analogous to the federal law, beyond filing and timing requirements. The state court specified that it made its ruling in light of its interpretation of federal precedent regarding the federal law (the Americans with Disabilities Act). Therefore, the Supreme Court may not desire to hear the petition. However, in light of the state court’s reliance on federal precedent, the Supreme Court may desire to grant the petition and make a review of the matter, as it is certainly the final arbiter of the interpretation of federal precedent. Therefore, this answer is correct.

189
Q

Abstention while waiting for the state court

A

The abstention doctrine permits the federal court to temporarily refuse jurisdiction over a federal constitutional issue if it is possible that the state court will dispose of the issues relying on unsettled state law. There is a federal constitutional issue presented in the case, because the town’s refusal to translate its upcoming ballot for the benefit of citizens not fluent in English may violate the Equal Protection Clause by discriminating against individuals on the basis of race or national origin and by infringing on the fundamental right to vote. However, the state court hearing the town’s case may resolve the issue without requiring the federal court to reach the constitutional issue. For example, if the state court rules that the ballot in the upcoming district vote must be translated despite time constraints, then the federal issue need not be reached.

190
Q

No sovereign immunity for subdivisions of the state

A

The Eleventh Amendment prohibits the citizens of one state from suing their own state or another state on federal claims for money damages, in federal court or state court, without the state’s consent. Eleventh Amendment immunity applies not only to diversity suits but to federal question cases as well. The concept of governmental, or sovereign, immunity means that the government may not be sued without its consent. However subdivisions of a state (such as cities, towns, and counties) do not have immunity from suit under the Eleventh Amendment.